Публикация научных статей.
Вход на сайт
E-mail:
Пароль:
Запомнить
Регистрация/
Забыли пароль?

Научные направления

Поделиться:
Разделы: Астрономия
Размещена 07.12.2022.
Просмотров - 3571

Какие тела падают быстрее лёгкие или тяжёлые?

Дудин Александр Тимофеевич

нет

не работаю

пенсионер

Аннотация:
В настоящее время утверждается, что ускорение свободного падения не зависит от массы падающих тел. Это утверждение перенесено в космос, что вносит ошибку в расчёты, вызывает потребности в поиске тёмной материи, непонимания в закономерности расположения планет.


Abstract:
Annotation. Currently, it is claimed that the acceleration of free fall does not depend on the mass of falling bodies. This statement has been transferred to space, which makes an error in calculations, causes the need to search for dark matter, misunderstandings in the regularities of the location of planets.


Ключевые слова:
ускорение свободного падения; масса; планета; орбита; тела

Keywords:
acceleration of gravity; mass; planet; orbit; bodies


УДК 52, 53

Введение.

Ускорением свободного падения интересовались с глубокой древности. Философ  Аристотель утверждал, что тяжёлые тела падают на Землю быстрее, чем лёгкие. И это утверждение было принято за аксиому. Впервые это утверждение было опровергнуто Галилео Галилеем на основе проведённых экспериментов. Проводя опыты, при одновременном сбрасывании тел из разного вещества с Пизанской башни, и катая шары по наклонным плоскостям под разными углами, он пришёл к выводу, что тела разной массы падают с одинаковым ускорением. Что ускорение падающих тел не зависит от массы падающих тел, а зависит от характеристики Земли.

И.Ньютон изготовил стеклянную трубку, в которую поместил перо и золотую монету и откачал воздух, что наглядно демонстрирует, что перо и монета падают одновременно.

Это утверждение сохраняется по сегодняшний день. Проведено много научных экспериментов и во всех экспериментах это утверждение продолжает подтверждаться.

В формуле тяготения И.Ньютона:

F = G Mm/R^2, при определении ускорения масса малого тела сокращается: g = GM/R^2.

Физик Брайан Кокс в вакуумной камере HASA провёл эксперимент с падением  перьев и металлического шара, покрытого пластмассой, и подтвердил одновременность падения тел.

При написании работы опирался на следующие источники: [1];[2];[3];[4].

Актуальность данной работы обусловлена интенсивным освоением космоса, развитием физики, астрофизики, астрономии, космологии, изучением взаимодействия звёздных систем, галактических взаимодействий, скопления галактик.

Цели и задачи данной работы заключаются в том, чтобы определить в космосе какие тела падают на центральное тело быстрее: лёгкие или тяжёлые? 

Научная новизна данной работы, заключается в том, что проведя сравнительные расчёты, приводится доказательство, что лёгкие тела падают быстрее тяжёлых тел. 

В интернете есть много размышлений на тему: «Две планеты на одной орбите?»,- выводы разные и не обоснованные, не подкреплённые расчётами, так, гадание на кофейной гуще.

В данной работе проведём свои расчёты и покажем что, это подвергается расчётам, получаем обоснованные выводы.

 

Определим, может ли Венера находиться с Землёй на одной орбите?

М(С) = 1,9885*10^30 кг

М(З) = 5,9726*10^24 кг

М(В) = 4,8675*10^24 кг

R(З б.п.) = 149 261598  км
G = 6,67 * 10^-11 м^3 /кг с^2

 

Находим силу воздействия Солнца на Венеру:

F (C-B) = G M(С) M(В) /R^2

F (C-B) = (6,67 * 10^-11 м^3 /кг с^2 *1,9885*10^30 кг*4,8675*10^24 кг) /

(1,49261598*10^11 м)^2 = 64,5590884125 *10^43 м^3 кг/ с^2 / 2,22790246375136*10^22 м^2 = 28,97752009475086 *10^21 м кг/ с^2

F (C-B) = 28,97752009475086 *10^21 м кг/ с^2                   (1)

 

Находим силу воздействия Солнца на Землю:

F (C-З) = G M(С) M(З) / R^2

F (C-З) = (6, 67 * 10^-11 м^3 /кг с^2 *1,9885*10^30 кг*5,9726*10^24 кг) /

(1,49261598*10^11 м)^2 = 79,21635571699998 *10^43 м^3 кг/ с^2 / 2,22790246375136*10^22 м^2 = 35,55647386089553 *10^21 м кг/ с^2

F (C-З) = 35,55647386089553 *10^21 м кг/с^2                     (2)

 

Находим относительно Солнца  ускорение Венеры.    

F (C-B) = 28,97752009475086 *10^21 м кг/ с^2

F (C-B) = М(В) g(В) 

g(В) = F (C-B) / М(В) = 28,97752009475086 *10^21 м кг/ с^2 / 4,8675*10^24 кг = 5,953265556189185 *10^-3 м / с^2

g(В) = 5,953265556189185 *10^-3 м / с^2                              (3)

 

Находим относительно Солнца ускорение Земли.

F (C-З) = М(З) g(З)   

g(З) = F (C-З) / М(З) = 35,55647386089553 *10^21 м кг / с^2 / 5,9726*10^24 кг = 5,953265556189185 *10^-3 м / с^2

g(З) = 5,953265556189185 *10^-3 м / с^2                               (4)

 

Как видим ускорение Земли и ускорение Венеры относительно Солнца одинаковое, и равно:

g(В) = g(З) = 5,953265556189185 *10^-3 м / с^2                    (5)

 

К сожалению, во всех случаях на этом расчёт ускорений заканчивается.

В соответствии с третьим законом И. Ньютона и формулой Всемирного тяготения,

согласно общепринятому утверждению, что Солнце притягивает планеты, и планеты притягивают Солнце, рассчитаем ускорение Солнца относительно Венеры:

g(C-B) = F(C-B) / М(С) = 28,97752009475086 *10^21 м кг/ с^2 / 1,9885*10^30 кг

 = 14,5725522226557 *10^-9 м / с^2

g(C-B) = 14,5725522226557 *10^-9 м / с^2                             (6)

 

Находим ускорение Солнца относительно Земли:

g(C-З) = F(C-З) / М(C) = 35,55647386089553 *10^21 м кг/ с^2 / 1,9885*10^30 кг = 17,88105298511216 *10^-9 м / с^2

g(C-З) = 17,88105298511216 *10^-9 м / с^2                            (7)

 

Как видим, в соответствии с расчётом, ускорение Солнца к Венере и ускорение Солнца к Земле разные: (6) и (7).

 

В данный момент подошли к самому трудному моменту в понимании, так как он всегда игнорировался при преподавании во всех учебных заведениях и в учебниках.

Исходя из того, что инерционная масса равна гравитационной массе, гравитационную силу надо рассматривать, как инерционную, то есть силу сопротивления, поэтому ускорения направлены по полю к соответствующим телам, а не наоборот.

Трудно представить, что Солнце может сдвинуть планета Меркурий или Земля. Они Солнце и не сдвигают, просто планеты, находясь в постоянном напряжении взаимодействия, имея разную инерционную массу, сопротивляются подтягиванию самих себя к Солнцу. Солнце сообщает телам одинаковые ускорения, находящимся на одной орбите, за счёт того, что тела имеют разную массу, тела притягивают Солнце с разным ускорением, но так как Солнце настолько массивное, то эти разные ускорения передаётся самим телам в противоположном направлении, которые и падают с разной скоростью.

Это утверждение исходит из третьего закона И.Ньютона.

M(1)a(1) = M(2)a(2)

Силы взаимодействия двух тел равны и противоположно направлены, следовательно, их ускорения направлены в противоположные стороны.

 

Поэтому результирующее ускорение для Венеры находим:

g(рез.B) = g(В) - g(C-B)

g(рез.B) = 5,953265556189185 *10^-3 м - 14,5725522226557 *10^-9 м / с^2 =

5953265,556189185 *10^-9 м - 14,5725522226557 *10^-9 м / с^2 = 5953250,983636962*10^-9 м / с^2

g(рез.B) = 5953250,983636962*10^-9 м / с^2                           (8)

                            

Результирующее ускорение для Земли находим:

g(рез.З) = g(З) - g(C-З)

g(рез.З) = 5953265,556189185 *10^-9 м - 17,88105298511216  *10^-9 м / с^2 = 5953247,6751362*10^-9 м / с^2

g(рез.З) = 5953247,6751362*10^-9 м / с^2                                 (9)

 

Находим насколько ускорение Венеры к Солнцу больше, чем ускорение Земли к Солнцу:

g(рез.B) -  g(рез.З) = 5953250,983636962*10^-9 м / с^2 -  5953247,6751362*10^-9 м / с^2 = 3,308500762*10^-9 м / с^2

g(рез.B) -  g(рез.З) = 3,308500762*10^-9 м / с^2

 

Ускорение Венеры к Солнцу больше, чем ускорение Земли к Солнцу на одном радиусе с Землёй. Следовательно, тела, имеющие меньшую массу, падают быстрее на центральное тело, и на одной орбите находиться не могут. На одной орбите могут находиться только две планеты, имеющие одинаковые массы. Но в природе такой феномен вряд – ли может быть, так как планеты формируются миллиарды лет. А если и есть, то это уникальный случай, так как, даже с небольшой разницей в массе, Венера и Земля отстоят друг от друга на значительном расстоянии.

 

Почему притягивает большая масса меньшую массу? Потому, что одинаковые массы не притягиваются. Так как у них одинаковые ускорения, направленные в разные стороны.

Поэтому Г.Кавендиш, взяв разные массы для эксперимента, сделал всё правильно.

Заключение.

Следовательно, лёгкие тела с одинаковой орбиты падают быстрее тяжёлых тел на центральное тело, и на одной орбите находиться не могут.

Поэтому орбиты Меркурия, Венеры, Земли расположены  в зависимости от массы.

Перестройка планет в зависимости от массы и расстояния продолжается и сегодня

За многие миллионы лет планеты перестроились в соответствии с массами, и влияния внутреннего и  внешнего гравитационного воздействия. Перестройка планет по орбитам, в соответствии с массами, приводила к столкновению космических тел.

Выводы. Актуальность работы очевидна. Цели и задачи работы выполнены.

Научная новизна данной работы заключается в том, что проведя сравнительные расчёты, приводится доказательство, что лёгкие тела падают быстрее тяжёлых тел.

Это в корне меняет расчёт взаимодействия космических тел, расчётов, как в галактике, так и галактических скоплений. Отменяется поиск тёмной материи.

Библиографический список:

1. Свободное падение — Википедия /электронный ресурс/ https://ru.wikipedia.org/wiki/Свободное_падение (дата посещения: 05.12.2022 г.)
2. Эксперименты Галилея по падению тел — Википедия /электронный ресурс/ https://ru.wikipedia.org/wiki/Эксперименты_Галилея_по_падению_тел (дата посещения: 05.12.2022 г.)
3. Эксперимент с падением двух различных тел в вакууме /электронный ресурс/ https://fishki.net/video/1324497-jeksperiment-s-padeniem-dvuh-razlichnyh-tel-v-vakuume.html (дата посещения: 05.12.2022 г.)
4. Спросите Итана: могут ли две планеты существовать на одной орбите? / Хабр /электронный ресурс/
https://habr.com/ru/post/400411/ (дата посещения: 05.12.2022 г.)




Рецензии:

12.11.2023, 9:04 Свирский Андрей Петрович
Рецензия: Наверное у некоторых читателей вызовет затруднение правильно ответить на вопрос, вынесенный в заголовок статьи. Однако тем, кто хорошо понимает курс школьной физики, ответить на него будет нетрудно - тяжелые тела при равных начальных условиях (отсутствия сопротивления воздуха, начальной скорости, одинаковой высоты падения и геометрии тела и пр.) при падении достигают Землю быстрее, чем лёгкие. Естественно, что приведённые в дальнейшем доказательства и вывод справедливы также для "падения" уже самой Земли, Венеры или других космических тел на Солнце. Обычно в школе ограничиваются констатацией того факта, что все тела, независимо от их массы, падают на Землю с одинаковым ускорением "g" (что конечно правильно), не акцентируя внимания на один нюанс. А именно на то, что при этом одновременно происходит падение не только тела на Землю, но и в соответствие с законом всемирного тяготения, падение Земли на тело. Поэтому здесь правильнее вместо термина "падение тела на Землю" использовать термин "сближение тела и Земли". Такое взаимное сближение приводит к тому, что путь до соприкосновения с Землёй у более тяжелого тела становится короче, оно больше как бы "вытягивает" Землю на себя, чем легкое тело. И хотя на всей дистанции сближения, благодаря равенству гравитационной и инерционной масс, ускорения и скорости падающих тяжелых и легких тел не будут отличаться между собой, более короткий путь приведёт к тому, что тяжелое тело достигнет Землю быстрее. Сделанный автором в статье противоположный вывод, что лёгкие тела падают (сближаются) быстрее тяжёлых - ошибочен. Соответственно и следствия, выведенные на основании ошибочного вывода, необоснованы. Статью к публикации не рекомендую.

12.11.2023 14:14 Ответ на рецензию автора Дудин Александр Тимофеевич:
Уважаемый Андрей Петрович! Мне очень понравилась Ваша рецензия, Вы, рассуждая, доводя до сведения, высказали своё мнение. Ценнее этого быть не чего не может. Вы очень правильно отметили взаимодействие тел. И, неважно, измените Вы рецензию или нет, приведу свои возражения. Начнём с аналогии. Есть две команды, допустим А и Б, по перетягиванию каната. По аналогии каждая команда, это космическое тело. Канат, - это гравитационная связь между телами. Земля или пол, или площадка бетонная, - это среда, аналогична вакууму. Вакуум заполнен космическими частицами, нейтрино и другими, не случайно, скорость света определяют через магнитную и диэлектрическую проницаемости. Вакуум, - это среда. Участники по перетягиванию каната равные по возрасту и по силе. На одну сторону каната становиться команда А из 10 человек, а на другую сторону на перетягивание каната выходит команда Б - один человек, и хотя он упирается, как может, команда А из 10 человек его перетягивает. Здесь силы не равны и ускорения направлены в разные стороны. После каждого поражения, команда Б добавляет по одному участнику. С добавлением участника в команду Б, команда А чувствует всё большее и большее сопротивление. И с выходом команды Б в количестве 10 человек, силы команд становятся равными, ускорения направлены в разные стороны и равны между собой, команды не могут перетянуть друг друга. А вот теперь, если к команде Б присоединяется ещё один участник, то команда Б перетягивает команду А. Всё аналогично происходит и с космическими телами. Приведём пример с космическими телами. Возьмём космическое тело, равное по массе Солнцу и два тела равные по массе Меркурию и Юпитеру допустим на расстоянии большой полуоси Юпитера. Масса тела С = 1,9885*10^30 кг Масса тела М = 3,33022*10^23 кг Масса тела Ю = 1,8986*10^27 кг Тела М и Ю находятся на расстоянии от тела С, равном 7,785472*10^11м G = 6, 67430 *10^-11 м^3/ кг* c^2 Находим по ЗВТ И. Ньютона ускорение тела М к телу С. F = G*М(C)*m(M) / R^2 m(M)*а(M) = G*М(C)*m(M) / R^2 а(M) = G*М(C) / R^2 а(M) = 6, 67430 *10^-11 м^3/ кг* c^2 *1,9885*10^30 кг / (7,785472*10^11м)^2 = 0,2189583061454396 *10^-3 м/с^2 а(M) = 0,2189583061454396 *10^-3 м/с^2 Находим по ЗВТ И. Ньютона ускорение тела Ю к телу С. F = G*М(C)*m(Ю) / R^2 m(Ю)*а(Ю) = G*М(C)*m(Ю) / R^2 а(Ю) = G*М(C) / R^2 а(Ю) = 6, 67430 *10^-11 м^3/ кг* c^2 *1,9885*10^30 кг / (7,785472*10^11м)^2 = 0,2189583061454396 *10^-3 м/с^2 а(Ю) = 0,2189583061454396 *10^-3 м/с^2 а(M) = а(Ю) Как видим ускорение тела М и ускорение тела Ю к телу С совершенно одинаковые. И это было бы так, если бы действовало, только тело С, на тела М и Ю, то есть массы тел М и Ю для тела С не чего не значили и на тело С не какого воздействия не оказывали. Но закон ЗВТ И.Ньютона говорит о взаимодействии тел. Поэтому найдём как тела М и Ю воздействуют на тело С. Находим по ЗВТ И. Ньютона ускорение тела С к телу М. F = G*М(C)*m(M) / R^2 М(С) а(С) = G*М(C)*m(M) / R^2 а(С) = G*m(M) / R^2 а(С) = 6, 67430 *10^-11 м^3/ кг* c^2 * 3,33022*10^23 кг / (7,785472*10^11м)^2 = 0,3666981796789872 *10^-10 м/с^2 а(С) = 0,3666981796789872 *10^-10 м/с^2 Находим по ЗВТ И. Ньютона ускорение тела С к телу Ю. F = G*М(C)*m(Ю) / R^2 М(С) а(С) = G*М(C)*m(Ю) / R^2 а(С) = G*m(Ю) / R^2 а(С) = 6, 67430 *10^-11 м^3/ кг* c^2 *1,8986*10^27 кг / (7,785472*10^11м)^2 = 0,2090592104841497*10^-6 м/с^2 а(С) = 0,2090592104841497*10^-6 м/с^2 Как видим, ускорение тела С к телу М на 4 порядка меньше, чем к телу Ю. Большая масса тела Ю вызывает большее ускорение тела С. То есть тело Ю больше сопротивляется телу С в перемещении через среду, так как у него больше масса, чем у тела М. Исходя из вышесказанного определим ускорения сближения тел С и М и тел С и Ю. а(М - С) = 0,2189583061454396 *10^-3 м/с^2 - 0,3666981796789872 *10^-10 м/с^2 а(Ю - С) = 0,2189583061454396 *10^-3 м/с^2 - 0,2090592104841497*10^-6 м/с^2 Отсюда очевидно, что а(М - С) больше а(Ю - С) Вывод: лёгкие тела падают быстрее, чем более тяжёлые тела с одного расстояния на одно и то же центральное тяжёлое тело, и это очень важно для расчёта космических тел. С уважением А.Т. Дудин.

13.11.2023, 11:32 Свирский Андрей Петрович
Рецензия: Уважаемый Александр Тимофеевич! Все тела притягиваются друг к другу, что сказывается на траектории их движения. Оценить всё это в полной совокупности конечно невозможно, и Вы правильно сделали, ограничившись для примера лишь рассмотрением взаимодействия между двумя телами. Правильна ли Ваша аналогия с двумя командами? Думаю, что нет. Чем объясняется гравитация и какой механизм взаимодействия работает между удалёнными друг от друга телами мы до сих пор совершенно не понимаем, можем только достоверно и с большой точностью определить величины сил, которые возникают в результате этого взаимодействия (по закону ЗВТ). То есть сила, которую развивает команда А, прикладывается к объекту Б не через канат (это важно!), а каким-то образом непосредственно. Повторюсь, никто не знает почему это происходит, но это происходит именно так. Никаких действий и противодействий согласно третьему закону И. Ньютона здесь нет, в отличие от случая, если бы усилие передавалось через канат. Аналогичным образом и команда Б прикладывает свою силу к объекту А не через канат, а непосредственно. Ускорение от приложенной к объекту Б силы направлено к объекту А и наоборот. То есть образующиеся ускорения всегда складываются, независимо от масс взаимодействующих тел. И чем массивнее будут тела, тем быстрее они будут сближаться, так как дистанция между ними будет укорачиваться вследствие большего "вытягивания" их друг на друга. С уважением, А.П.
13.11.2023 15:15 Ответ на рецензию автора Дудин Александр Тимофеевич:
Уважаемый Андрей Петрович! Закон ЗВТ Вы противопоставляете третьему закону И. Ньютона? Давайте посмотрим, что это такое? Возьмём Солнце и Землю и запишем их взаимодействия. F = G*М(C)*m(З) / R^2 (1) m(З)а(З) = G*М(C)*m(З) / R^2 (2) m(З)а(З) = G*М(C)*m(З) / R^2 (3), сокращаем: а(З) = G*М(C) / R^2 (4) М(C)а(С) = G*М(C)*m(З) / R^2 (5), сокращаем: а(С) = G*m(З) / R^2 (6), Из уравнений (4) и (6) выведем гравитационную постоянную: а(З) = G*М(C) / R^2 (4) G = а(З)* R^2 / М(C) (7) а(С) = G*m(З) / R^2 (6), G = а(С)* R^2 / m(З) (8) Так как гравитационная постоянная у нас одна и та же, то правые части уравнений (7) и (8) будут так же равны, как и левые. G = а(З)* R^2 / М(C) (7) G = а(С)* R^2 / m(З) (8) а(З)* R^2 / М(C) = а(С)* R^2 / m(З) (9), сократим: а(З) / М(C) = а(С) / m(З) (10), приведём к виду: а(З)* m(З) = а(С)* М(C) (11) Как видим уравнение (11),- это третий закон И.Ньютона. То, что закон ЗВТ выведен из третьего закона И. Ньютона, это очевидно, более того гравитационная постоянная G является ликвидатором одной из сил третьего закона И. Ньютона. Вы правы, что все тела притягиваются друг к другу. Но в таком случае Вы вступаете в противоречие с самим собой. Тело, притягивая другое тело, преодолевает его инертную массу и сопротивление среды на всём пути перемещения. Суммируя ускорения тел, Вы, отрицаете значение инертной массы второго тела и сопротивление среды, Вы отрицаете воздействие одного тела на второе, так как в этом случае тело должно передвигаться под действием внешней силы. «То есть сила, которую развивает команда А, прикладывается к объекту Б не через канат (это важно!), а каким-то образом непосредственно. Повторюсь, никто не знает почему это происходит, но это происходит именно так». Свирский А.П. Андрей Петрович! Канат, это аналогия, как передаются силы от одного тела к другому, обменом частиц или через поле, или тем и другим способом, а может каналами связи между телами, в этой работе не рассматриваем. Поэтому не будем выходить за рамки работы. Прежде, чем окончательно написать рецензию, предлагаю Вам ознакомиться с работами указанными в моём отзыве в этой работе от 8.11.2023, 10:45 С уважением А.Т. Дудин.

14.11.2023, 10:06 Свирский Андрей Петрович
Рецензия: Уважаемый Александр Тимофеевич! У меня нет никаких возражений против Ваших преобразований формулы ЗВТ с получением финального соотношения а(З)* m(З) = а(С)* М(C) (1), схожего по написанию с третьим законом И.Ньютона. Однако вывод "То, что закон ЗВТ выведен из третьего закона И. Ньютона, это очевидно, более того гравитационная постоянная G является ликвидатором одной из сил третьего закона И. Ньютона" является ошибочным. Важно помнить (и понимать), что получение соотношения (1) стало возможным только при обязательном предварительном знании и согласии с ЗВТ (когда приравнивали члены с "G"). Можно ли утверждать обратное, что из ЗВТ выведен третий закон И. Ньютона, область действия которого распространяется и на удалённые друг от друга тела? На мой взгляд сомнительно. Ведь в этой области он действует только с некоторым условием, зависящем от другого закона, т.е ЗВТ. То, что будет, если его применить без учёта этого условия, наглядно показано в ошибочной статье "Гравитационная постоянная G, не постоянная", в которой "доказана" несостоятельность ЗВТ. Поэтому для достоверных расчётов взаимодействия удалённых друг от друга тел и во избежании грубых ошибок, лучше всегда применять только фундаментальный физический закон ЗВТ, хорошо проверенный экспериментально. Тогда не будет почвы для парадоксальных высказываний типа "Почему притягивает большая масса меньшую массу? Потому, что одинаковые массы не притягиваются. Так как у них одинаковые ускорения, направленные в разные стороны". На мой взгляд ЗВТ один из самых великих законов природы и, невзирая на простоту формулировки, самый таинственный. Ведь за прошедшие столетия наука так и не смогла ответить на вопрос, чем вызвана сила тяготения. Некоторые физики, например А. Эйнштейн, вообще взаимное тяготение тел объясняли совсем по-другому, не используя понятие силы. Но это я несколько отвлёкся. Вы пишите "Тело, притягивая другое тело, преодолевает его инертную массу и сопротивление среды на всём пути перемещения". Разумеется это так, в чём Вы увидели моё несогласие с этим? Очевидно, что силы, приложенные как к первому, так и ко второму телу, всё это должны преодолевать. Возможно мою мысль нужно было выразить почётче, например так - образующиеся ускорения направлены на сближение тел и чем больше их значения, тем тела быстрей сближаются. По поводу ознакомления с работами. Я недавно для себя открыл этот журнал. Некоторые статьи, в том числе и Ваши, меня заинтересовали. Надеюсь, что у меня будет время подробнее с ними ознакомиться. С уважением, А.П.
14.11.2023 21:21 Ответ на рецензию автора Дудин Александр Тимофеевич:
Уважаемый Андрей Петрович! «Галилей провел множество экспериментов, и его наблюдения привели его к выводу, что для каждого действия существует равное по величине, но противоположное по направлению действие. Это было важным прорывом в понимании законов природы, но Галилей не смог сформулировать закон точно» « В 1687 году Ньютон опубликовал свое главное произведение «Математические начала натуральной философии», в котором подробно описывался его закон действия и реакции. Согласно этому закону, когда одно тело оказывает силу на другое тело, второе тело оказывает равную по величине, но противоположную по направлению силу на первое тело». История открытия третьего закона ньютона | Электрофизик https://electrophysic.ru/pomosch/istoriya-otkrytiya-tret-yego-zakona-n-yutona.html «Некоторые ученые считали, что ученый промышляет плагиатом: мысль о том, что движение планет объясняется действием силы, которая притягивает каждую планету к Солнцу, уже высказывалась ранее, в том числе английским физиком Робертом Гуком — он даже сформулировал, что эта сила убывает обратно пропорционально квадрату расстояния от Солнца. Свою теорию Гук изложил в том самом 1666 году, когда на Исаака упало яблоко, а в 1679 году посылал Ньютону письмо, где предлагал сотрудничать по решению этой задачи, но получил отказ и заверения о том, что эта тема давно не занимает адресата. В дальнейшем Гук требовал указывать его имя как первого автора закона тяготения и открыто обвинял Ньютона в плагиате. Ученые конфликтовали до конца жизни Гука, а спор о том, кто был первым, продолжался даже в XX веке». Закон всемирного тяготения Ньютона - Телеканал "Наука" https://naukatv.ru/articles/819 «На основании 3-го закона механики Ньютон заключил, что притяжение есть взаимное свойство, и пришёл к формулировке своего закона тяготения для любых материальных ...» Ньютона закон тяготения ВОУНБ https://www.booksite.ru › fulltext «Закон всемирного тяготения и третий закон Ньютона не противоречат друг другу. По формуле (11.1) сила, с которой тело 1 притягивает тело 2, равно силе, с которой тело 2 притягивает тело 1.» Богданов К.Ю. - учебник по физике для 10 класса -§11 https://kaf-fiz-1586.narod.ru/10bf/uchebnik/11.htm#:~:text=Закон%20всемирного%20тяготения%20и%20третий,силой%2C%20равной%20весу%20капли%20дождя Генри Кавендиш не выводил гравитационную постоянную, он взвесил Землю. «Возможно впервые это было сделано французским физиком Пуассоном в «Трактате по механике» (1809), по крайней мере, никаких более ранних работ, в которых фигурировала бы гравитационная постоянная, историками не выявлено» Гравитационная постоянная — Википедия https://ru.wikipedia.org/wiki/Гравитационная_постоянная Закон всемирного тяготения записывается: F(1) = F(2) = GMm/R^2 Классическая теория тяготения Ньютона — Википедия https://ru.wikipedia.org/wiki/Классическая_теория_тяготения_Ньютона А разве это не третий закон И.Ньютона? И его варианты: F(1) = GMm/R^2; F(2) = GMm/R^2; F(1) = F(2); F(1) = F(2) = GMm/R^2 Андрей Петрович! Вы, оказывается, от чужого мнения сильно зависите? Тогда не будет почвы для парадоксальных высказываний типа "Почему притягивает большая масса меньшую массу? Потому, что одинаковые массы не притягиваются. Так как у них одинаковые ускорения, направленные в разные стороны". Казалось бы, несуразное высказывание, но очень удачное, так как только его все и запоминают. А чтобы разобраться в законе всемирного тяготения, вернитесь к аналогии с перетягиванием каната. Закон всемирного тяготения применяем односторонне, до сих пор не можем установить и измерить, что лёгкие тела падают быстрее тяжёлых тел, а это на космических просторах выливается в существенную ошибку. Гравитационные и центробежные силы применяем по отдельности, а они образуют единую систему взаимодействия, так как имеют разные зависимые законы изменения их от расстояния. Некоторые проблемы раскрыты в моих работах: Убегающие звёзды https://sci-article.ru/stat.php?i=1675158009 Гармония в угловых скоростях между планетами в Солнечной Системе https://sci-article.ru/stat.php?i=1655008723 Роль ядра Солнца и планет в перемещении тел по орбите вокруг себя https://sci-article.ru/stat.php?i=1655022445 Ускорения тел на орбите вокруг центрального тела. Часть 1 https://sci-article.ru/stat.php?i=1653847153 Ускорения тел на орбите вокруг центрального тела. Часть 2 https://sci-article.ru/stat.php?i=1653934276 Тайна закона всемирного тяготения https://sci-article.ru/stat.php?i=1613732883 Масса. - Физика - Новая Теория /электронный ресурс/ http://www.newtheory.ru/physics/massa-t6618.html С помощью простой арифметики всё просчитывается и доказывается на конкретных примерах без тёмной материи и тёмных сил на основании общеизвестных законов. С уважением А. Т. Дудин.

15.11.2023, 11:22 Свирский Андрей Петрович
Рецензия: Уважаемый Александр Тимофеевич! Мне кажется, что имеет смысл более подробно изложить свои соображения о приведённой Вами аналогии с командами А и Б. Допустим мы разместили стальной шарик (объект А) на стальную плиту (объект Б). В объекте А есть команда (сила тяжести) которая стремится продавить поверхность стальной плиты. В силу третьего закона И. Нютона (действие равно противодействию и направлено противоположно) в объекте Б появляется команда, равная по силе команде А, которая стремиться противодействовать перемещению шарика вглубь плиты. Если контакт шарика с плитой по каким-то причинам прервётся, то команда в объекте А останется прежней, а команда в объекте Б исчезнет. Как ситуация выглядит для удалённых друг от друга тел? Рассмотрим неподвижные (в начальный момент времени) тела, объекты 1 и 2. Согласно ЗВТ в них существуют команды А и Б, причём эти команды совершенно равны по отношению друг к другу. Нет того, что Вы рассматриваете в своём примере "На одну сторону каната становиться команда А из 10 человек, а на другую сторону на перетягивание каната выходит команда Б - один человек". Более того, между командами нет никаких сил противодействия. Команда А воздействует дистанционно на объект Б, стремясь притянуть его к объекту А, а команда Б аналогичным образом это проделывает с объектом А. Сила команд, и соответственно вызываемые ими ускорения объектов зависят от масс этих объектов. Так как величины сил у команд А и Б равны, то какой путь до сближения пройдёт каждый из объектов (и какое разовьёт ускорение) зависит только от их масс. Каким образом силы тяготения дистанционно прикладываются к объектам в настоящее время никто не знает. Возможно это какие-то свойства пространства или среды, их предполагаемая дискретность, что пытаются использовать для создания квантовой теории гравитации. Из изложенного видна разница между ЗВТ и третьим законом И. Ньютона в случае взаимодействия удалённых друг от друга тел. Надеюсь, что свои соображения изложил не очень сумбурно и смысл их понятен. С уважением, А.П.
19.11.2023 8:08 Ответ на рецензию автора Дудин Александр Тимофеевич:
Уважаемый Андрей Петрович! Рецензия пишется на статью, а не на примеры и аналогии, данные Вам для пояснения, что бы облегчить понимание сути. У Вас какие – то посторонние проблемы, не касающиеся статьи? Структура статьи выполнена в соответствии с требованиями. На оригинальность и плагиат работа прошла проверку модераторами. Актуальность темы, куда уж актуальней? Научная новизна данной работы, заключается в том, что проведя сравнительные расчёты, приводится доказательство, что лёгкие тела падают быстрее тяжёлых тел. Какие у Вас претензии к расчётам? Вы нашли ошибку в расчётах и её следует исправить? Расчёты выполнены на основании закона всемирного тяготения И.Ньютона (ЗВТ), и не каких отклонений от этого нет, какие претензии? «Каким образом силы тяготения дистанционно прикладываются к объектам в настоящее время никто не знает», - вот это зачем и какое отношение имеет к статье? Если Вы говорите, что тела взаимодействуют, но при этом запрещаете провести расчёт по ЗВТ с двух сторон, то это очень странный подход рецензента, очень «научный»? Если Вы не допускаете новый взгляд, потому, что Вы его не понимаете или не разделяете, то подумайте, как А. Эйнштейн мог написать свои работы? Рецензия пишется корректно, доброжелательно, чтобы помочь автору продвинуть новые взгляды, а не тормозить их, потому что «до этого не было, и вот опять» или исходя из того, что « я начальник, ты дурак, ты начальник, я дурак». Если не согласны с расчётами, представьте свои и докажите обратное, или укажите на ошибку, а так огульно говорить, что этого не может быть не когда, потому, что не кто не знает, что такое гравитация, по крайней мере вызывает недоумение? А если кто – то знает и скажет своё мнение, то, как Вы ему рецензию напишите? Эту работу и без Вашей рецензии не печатали, что же Вы так обеспокоились, чтоб её не напечатали? Если есть повод для такого волнения, то надо доказать это в рамках статьи. С уважением А.Т. Дудин.

15.11.2023, 14:17 Свирский Андрей Петрович
Рецензия: Уважаемый Александр Тимофеевич! К сожалению я своё дополнение к рецензии от 15.11.2023, 11:22 написал раньше, чем увидел Ваш опубликованный ответ от 14.11.2023 21:21 на более раннюю публикацию. Если в целом, то всё что я сказал ранее, считаю правильным. Может только стоит прокомментировать следующие Ваши высказывания. "Генри Кавендиш не выводил гравитационную постоянную, он взвесил Землю" - это очень броское, расхожее и популярное мнение, которое любят приводить в общедоступных медиаресурсах. Нельзя сказать что оно по смыслу неверно, но для развития науки более важным оказалось то, что этот эксперимент в конечном итоге позволил получить современную запись формулы ЗВТ, а не результат взвешивания единичного тела. С тех пор были повторены сотни подобных экспериментов и в научной среде их никто и никогда не называл взвешиванием Земли. Насчёт фразы "То, что казалось бы, несуразное высказывание, но очень удачное, так как только его все и запоминают". Неужели она может характеризовать автора с положительной стороны только из-за того, что она хотя и неправильная, но запоминающаяся? Возражений к выписке из учебника К.Ю.Богданова "...сила, с которой тело 1 притягивает тело 2, равно силе, с которой тело 2 притягивает тело 1.» у меня нет. В своём дополнении я об этом написал подробно. Там же подробно написал, в чём вижу принципиальную разницу между третьим законом И. Ньютона и ЗВТ при взаимодействии удалённых друг от друга тел. Если Вы с этим не согласны, то попробуйте вывести ЗВТ из третьего закона И. Ньютона, но при этом исходите из того, что о ЗВТ Вам ничего не известно. Уверен, что у Вас этого не получится и фраза "То, что закон ЗВТ выведен из третьего закона И. Ньютона, это очевидно ...." совершенно ошибочна. Так что пока Вы меня не убедили, что вывод в статье не ошибочный. С уважением, А.П.
19.11.2023 8:08 Ответ на рецензию автора Дудин Александр Тимофеевич:
Уважаемый Андрей Петрович! Вы простите меня, давайте не будем заниматься демагогией и тратить время попусту. К этой статье, какое имеет отношение, как выведено ЗВТ, и какое отношение оно имеет к третьему закону И. Ньютона? Нет не какого противопоставления ЗВТ третьему закону И.Ньютона? Я где – то нарушил ЗВТ или применил его неправильно? Или в статье применил ЗВТ и третий закон в противопоставлении? У Вас какие – то необоснованные требования к статье, так можно дойти, что надо сначала доказать теорию большого взрыва. Боитесь ответственности, написать положительную рецензию, но этого некто не просит, это ваше личное дело. Взялись написать отрицательную рецензию, аргументируйте, докажите ошибки, если аргументом является другой взгляд, то это очень странный подход? Тем более, в статье представлены расчёты и доказательства. Как же Вы науку продвигать собираетесь или верны, только ваши взгляды, а другие взгляды не допустимы? С уважением А.Т. Дудин

25.11.2023, 12:05 Свирский Андрей Петрович
Рецензия: Уважаемый Александр Тимофеевич! Согласен, что не стоит больше попусту тратить время на обсуждение ошибочного вывода и утверждений в данной статье. Своё отрицательное мнение о ней я достаточно подробно обосновал ранее. Просто заинтересовало, чем было продиктовано Ваше утверждение, что закон ЗВТ выведен из третьего закона И. Ньютона. К сожалению ничем и, более того, закон всемирного тяготения Вы похоже совсем не понимаете, если утверждаете, что тела с одинаковой массой не притягиваются друг к другу. С уважением, А.П.
28.11.2023 4:04 Ответ на рецензию автора Дудин Александр Тимофеевич:
Уважаемый Андрей Петрович! Мне жаль, что Вы в своей рецензии опираетесь на огульный вывод, игнорируя расчёты в статье и все ответы на ваши рецензии. По необоснованной причине, Вам нужно «чудо» вывода закона всемирного тяготения? Напомню Вам, что историками установлено, гравитационная постоянная впервые появилась в 1809 г. Гравитационная постоянная — Википедия https://ru.wikipedia.org/wiki/Гравитационная_постоянная Вывод ЗВТ Имеем третий закон И.Ньютона Ма(1) = m a(2) Делим обе части уравнения на расстояние R^2 между телами: Ма(1)/R^2 = ma(2)/R^2 Силу взаимодействия двух тел запишем: W= (Ма(1)/R^2) * (ma(2)/R^2) W = (Ма(1)/R^2) * (ma(2)/R^2) = (а(1)* a(2)) * M*m / R^4 Обозначим: (а(1)* a(2)) = k, Запишем: W = k * M*m / R^4 (W)^1/2 = (k * M*m / R^4)^1/2 Преобразуем так, чтобы выражение (k * M*m)^1/2 было равно после извлечения корня: (k * M*m)^1/2 = G M*m, запишем формулу ЗВТ после преобразования: F = G M*m / R^2 «Просто заинтересовало, чем было продиктовано Ваше утверждение, что закон ЗВТ выведен из третьего закона И. Ньютона. К сожалению ничем и, более того, закон всемирного тяготения Вы похоже совсем не понимаете, если утверждаете, что тела с одинаковой массой не притягиваются друг к другу. С уважением, А.П.» Скажите, какое это имеет отношение к статье? Вы понимаете и игнорируете закон ЗВТ, рассматриваете его односторонне. Во взаимодействии участвуют два тела, а не одно тело. Ещё раз Вам объясняю, что тела с одинаковой массой взаимодействуют друг с другом, но их взаимодействия равны, то есть силы равны, приложены к разным телам и направлены в разные стороны, поэтому они не могут (сближаться друг с другом) притягиваться друг к другу. Это как две команды по перетягиванию каната, когда силы равны, то перетянуть не одна команда не может другую. Вы исключаете из взаимодействия одно из тел, при одинаковой массе, какое тело исключаете из взаимодействия? Или оба тела должны приталкиваться друг к другу внешними силами, а это не взаимодействие и не закон ЗВТ. С уважением А.Т. Дудин.



Комментарии пользователей:

10.12.2022, 18:42 Нечаев Алексей Вячеславович
Отзыв: Уважаемый Александр Тимофеевич! Считаю, что тема, поднятая Вами в статье, интересна для многих читателей и статья должна быть опубликована. С уважением!


11.12.2022, 6:36 Дудин Александр Тимофеевич
Отзыв: Уважаемый Алексей Вячеславович! Спасибо за отзыв. Есть надежда, что на эту проблему обратят внимание. С уважением А.Т. Дудин.


24.12.2022, 21:55 Дудин Александр Тимофеевич
Отзыв: Уважаемый Улугбек Товфикович! Спасибо за рецензию. Поясню и приведу дополнительные доказательства. Понятие тёмной материи возникло на несоответствии кеплеровской механики, согласно её скорость звезд при удалении от центра галактики должна уменьшаться. Но звёзды в центре и на периферии спиральных галактик движутся примерно одинаково В рукавах спиральных галактик образуются скопления звёзд, по ходу движения впереди и сзади образуются структуры из звёзд и газа, в которых звёзд должно быть примерно одинаково, но во всех случаях в передней части рукава звёзд больше, чем в задней части рукава за скоплением звёзд. Вроде бы в соответствии с законом тяготения И.Ньютона в каком месте окажется звезда, дело случая? Объяснить этот факт с помощью темной матери оказалось очень проблематично. Оказалось, что объяснить это явление можно с помощью модели MOND. Эта модель предполагает, что ускорение нелинейно и для слабых ускорений гравитация сильнее, чем это следует из закона И. Ньютона. Загадочное движение звезд. Физики поставили под сомнение теорию гравитации - РИА Новости, 15.11.2022 https://ria.ru/20221115/gravitatsiya-1831428617.html В моей работе заявление об отмене поиска тёмной материи вполне правомерно, так как, в соответствии с расчётами, доказано, что более лёгкие тела падают быстрее тяжёлых тел на центральное тело. Если в модели MOND, ещё не доказано, что для слабых ускорений гравитация сильнее, чем это следует из закона И. Ньютона, а заявление сделано? Во всех работах, где ищется отличие от закона тяготения И. Ньютона, как следствие, отменяется поиск тёмной материи. Смысл поиска её пропадает. Швейцарские ученые опровергли темную материю? - SWI swissinfo.ch https://www.swissinfo.ch/rus/sci-tech/космология_швейцарские-ученые-отменили-темную-материю-/44128400 Динамические эффекты масштабной инвариантности пустого пространства: падение темной материи? - IOPscience https://iopscience.iop.org/article/10.3847/1538-4357/aa92cc Выдвинута новая теория гравитации, альтернативная темной материи | Атомная энергия 2.0 https://www.atomic-energy.ru/news/2022/07/12/126287 Неожиданно: новые данные подтверждают модифицированную теорию гравитации | Пикабу https://pikabu.ru/story/neozhidanno_novyie_dannyie_podtverzhdayut_modifitsirovannuyu_teoriyu_gravitatsii_7953546 В соответствии с законом тяготения И. Ньютона, каждый объект создаёт гравитационное поле, сила притяжения распространяется на любые расстояния и действует между любыми телами. Закон тяготения И.Ньютона применялся, игнорируя ускорения, создаваемые малыми телами, так и писали: «…ими можно пренебречь». До «пренебрегались», выйдя на уровень галактик, потребовалась тёмная материя. В рукавах спиральных галактик из скоплений звёзд, звёзды с меньшей массой перемещаются в переднюю часть рукава, а звёзды большей массы перемещаются в заднюю часть рукава. Но так как звёзд в передней части рукава становиться больше, и масса их больше, чем в задней части рукава, то скорость рукава в передней части замедляется. Поэтому скорости звёзд в рукаве выравниваются. Земля сообщает всем падающим телам одинаковое ускорение, не зависимо от их массы. А каждое падающее тело в зависимости от массы сообщает своё ускорение Земле. Разница в массах пробных тел настолько мала, что эти ускорения заметить на глаз невозможно, речь должна идти о пробных телах, отличающихся в массе на порядки, а точность измерения должна быть 10^- 20 и выше. Такую точность измерений проблематично обеспечить даже лазерным лучом. Осталось провести только расчёты и принять их к сведению. Эксперимент с падением дробинки, кусочка резины и пёрышка в метровой вакуумной трубке с визуальным наблюдением заведомо обречён на обман. Все массы измеряются в граммах, то какую разницу в падении можно обнаружить? Эксперимент без расчёта это не эксперимент. А утверждать об одинаковом ускорении падающих тел с разной массой, это отрицать взаимодействие тел. Это равносильно утверждать, что нуклоны большего тела притягивают нуклоны меньшего тела, при этом нуклоны меньшего тела на большее тело не действуют. Для расчёта возьмём пробное тело m(1) = 1 гр = 0,001 кг = 10^-3 кг и пробное тело m(2) = 10 кг Масса Земли: M = 5,9726 *10^24 кг Средний радиус Земли: r = 6371,0 км Высота сбрасывания груза над Землёй = 10 м R = r + h = 6371000 м + 10 м = 6371010 м = 6,37101 * 10^6 м G = 6,67430 *10^-11 м^3 / кг с^2 По формуле тяготения И. Ньютона определим ускорения пробных тел Землёй. F(1) = G Mm(1) / R^2 F(1) = m(1) g(З) = G Mm(1) / R^2 g(З) = G M / R^2 g(З) = 6,67430 *10^-11 м^3 / кг с^2 * 5,9726 *10^24 кг / (6,37101 * 10^6 м)^2 = 0,9820929197580721 *10 м/с^2 g(З) = 9,820929197580721 м/с^2 С таким ускорением Земля притягивает пробное тело m(1) Находим ускорение, с которым Земля воздействует на пробное тело m(2) F(2) = G Mm(2) / R^2 F(2) = m(2) g(З) = G Mm(2) / R^2 g(З) = G M / R^2 g(З) = 6,67430 *10^-11 м^3 / кг с^2 * 5,9726 *10^24 кг / (6,37101 * 10^6 м)^2 = 0,9820929197580721 *10 м/с^2 g(З) = 9,820929197580721 м/с^2 Как видим, Земля не зависимо от массы любому пробному телу сообщает одинаковое ускорение: g(З) = 9,820929197580721 м/с^2 Находим, каким ускорением воздействует на Землю пробное тело m(1). F = G Mm(1) / R^2 F = M g(1) = G Mm(1) / R^2 g(1) = G m(1) / R^2 g(1) = 6,67430 *10^-11 м^3 / кг с^2 * 10^-3 кг / (6,37101 * 10^6 м)^2 = 1,644330642865874* 10^-26 м/с^2 g(1) = 1,644330642865874* 10^-26 м/с^2 Находим, каким ускорением воздействует на Землю пробное тело m(2). F = G Mm(2) / R^2 F = M g(2) = G Mm(2) / R^2 g(2) = G m(2) / R^2 g(2) = 6,67430 *10^-11 м^3 / кг с^2 * 10 кг / (6,37101 * 10^6 м)^2 = 1,644330642865874* 10^-22 м/с^2 g(2) = 1,644330642865874* 10^-22 м/с^2 g(1) = 1,644330642865874* 10^-26 м/с^2 Как видим, разница в ускорениях на 4 порядка, эти ускорения очень малы и на Земле они значения не имеют ими можно пренебрегать, но для космоса этот результат имеет огромное значение. Результирующее ускорение для лёгкого тела будет больше, чем для тяжёлого тела: g(З) - g(1) > g(З) - g(2) Лёгкие тела и на Земле падают быстрее тяжёлых тел, но этот факт не доступен для измерения. С уважением А.Т. Дудин.


31.12.2022, 8:48 Дудин Александр Тимофеевич
Отзыв: Уважаемый Улугбек Товфикович! Спасибо за отзывы, рекомендуемый и противопоставленный материал. Всё дело в том, что некоторые авторы пытаются поставить тёмную материю в основу, это выглядит так, что поставить телегу впереди лошади, при этом, не предлагая решения проблем с гравитационным взаимодействием. Тёмная материя введена от «несоответствия» вращения звёзд в рукавах галактик всемирному закону тяготения И. Ньютона. В моей работе показано, что закон тяготения И.Ньютона использован не в полной мере, открывается полнота применения закона И. Ньютона, при этом нет, не каких модернизаций закона. Все расчёты проведены, оспорить можно, только найдя ошибки в расчётах, или же продолжать утверждать, что центральное тело притягивает малое тело, а малое тело на центральное тело не оказывает взаимодействия. При этом продолжаем считать по закону тяготения И.Ньютона, но исключая взаимодействие малого тела? Данная работа направлена на полноту применения закона тяготения И. Ньютона, за пределами планеты, если на Земле ускорение падения тел разной массы «допустимо» считать одинаковым, то космические тела и системы рассчитывать с такой ошибкой не допустимо. В результате получаем совсем другие силы взаимодействия и ускорения тел. Дать рецензию на такой материал, большая ответственность, и мало, кто способен её взять на себя, не кому не хочется «сесть в лужу». В принципе на весь рекомендуемый и противопоставленный материал ответ дан, но чтобы не у кого не было сомнений пройдёмся по ссылкам: «1) Модифицированная ньютоновская динамика МОНД (Modified Newtonian dynamics) — это гипотеза, которая предлагает модификацию закона всемирного тяготения Ньютона для учета наблюдаемых свойств галактик [_https://en.wikipedia.org/wiki/Modified Newtonian_dynamics]». Короткий ответ. Гипотеза MOND исключает тёмную материю. «Большинство астрофизиков принимают темную материю Цвикки в качестве объяснения кривых вращения галактики…» Тёмная материя Цвикки, настолько проблематична, что брать её за основу в качестве объяснения, преждевременно. Цвикки ввёл понятие тёмная материя, строя расчёты массы галактик внутри скопления по их вращению, превышающее ожидаемое по их светимости в 400 раз. Проведя тот же самый расчёт сегодня, найдена существенная ошибка в показателях. Введя понятие тёмной материи, не было определено её свойств. Цвикки свои выводы сделал, наблюдая скопление галактик Кома в 1933 г. После его наблюдений и выдвинутой теории обнаружены сотни галактик в этом скоплении и на сегодня их количество более 1000. Каждая галактика имеет пылегазовые облака в поперечниках в несколько световых лет, мог их учесть Цвикке? Галактики окружены сильными магнитными полями, которые так же не учитывались при вращении. В настоящее время вокруг галактик обнаружено гало, в некоторых случаях превышающее размер галактики в 2 раза. Так что в тёмную материю по Цвикки вошла вся материя, которая была не видимой для него. В скоплении галактик Кома, находятся, как дисковые галактики, так и спиральные галактики, и к определению их масс подходить с одной меркой? Наблюдения Цвикки проводил в 1933 г., а границу скопления галактик Кома начали определять только в 1955 г. «[https://www.youtube.com/watch?v=UPVGDXNSBZM] на основе общей теории относительности и ньютоновской механики и привержены решению темной материи проблемы недостающей массы.Важнейшее подтверждение ОТО — экспериментальное обнаружение гравитационных волн, возникших при столкновении двух нейтронных звезд в 2016 году (Нобелевская премия мира по физике в 2017 г.)». Доктор Крупа рассматривает рождение звёзд, как рождение двойных систем, отрицает тёмную материю, утверждает, что эффективная гравитация должна быть неньютоновской в сверхслабых полях. «2) Ускорение свободного падения является важной геофизической величиной, которая определяется формой планеты и распределением масс в ней, скоростью вращения планеты вокруг своей оси, широтой местности. Ускорение свободного падения можно определить наиболее точным баллистическим методом (падение тела в вакууме), измеряемое интерферометрическими методами с относительной погрешностью порядка 10-7 – 10-6 или при помощи маятника Бесселя[Определение ускорения свободного падения при помощи маятника Бесселя : методические указания для выполнения лабораторных работ/Томск : Изд. ТГУ, 2022. – 12 с]. Любое тело, если его ничего не поддерживает, падает на поверхность Земли, постоянно [https://videouroki.net/video/06-svobodnoe-padenie-tel-dvizhenie-tela-pod-uglom-k-gorizontu.html], т.к. сила тяжести F = mg где: F — сила тяжести (Н); m — масса тела (кг); g — ускорение свободного падения (м/с²), на Земле g=9,81 м/с². Ускорение свободного падения на каждой планете солнечной системы имеет различную величину. Ускорение свободного падения равно: на Меркурии g=3,7 м/с², на Венере g=8,87 м/с², на Марсе g=3,711 м/с², на Юпитере g=24,79 м/с², на Сатурне g=10,44 м/с², на Уране g=8,69 м/с², на Нептуне g=11,15 м/с², на Нептуне g=0,658 м/с². Время, за которое тело достигнет поверхности той или иной планеты, зависит от ускорения свободного падения [https://lusana.ru/presentation/37467] и не зависит от массы падающего тела». Здесь нет, не чего нового, ускорение свободного падения, определяемое с помощью интерферометрических методов с точностью 10^-7 зафиксировать разницу ускорений свободного падения при разной массе не могут. В моем ответе от 24.12.2022 г. проведён расчёт, где точность измерения должна быть не менее 10^-22, а в работе выполнен расчёт ускорений свободного падения Земли и Венеры с одной орбиты на Солнце, то требуемая точность измерения должна быть не менее 10^-9. На основе справочных данных ускорения свободного падения на планеты, нельзя делать категорический вывод, что ускорение свободного падения от массы падающего тела не зависит. Поздравляю с Новым годом, желаю Здоровья, Благополучия, Новых Идей и Творческих Успехов. С уважением А.Т. Дудин.


28.02.2023, 19:10 Дудин Александр Тимофеевич
Отзыв: Греческий учёный и философ Аристотель утверждал, что тяжёлые тела падают быстрее, чем легкие. Он говорил, что камни падают быстрее, чем птичьи перья, кусок дерева падает быстрее, чем опилки, а это он объяснял, что чем больше предмет, тем быстрее он падает. Итальянский учёный Галилео Галилей понял, что Аристотель не учитывал сопротивление воздуха и провёл свои опыты по сбрасыванию шаров разной массы с Пизанской башни (подлинно не установлено) и катания шаров по наклонной плоскости. В результате проведённых опытов он пришёл к выводу, что тела разной массы из разных материалов падают с одинаковым ускорением. Дэвид Скотт астронавт Аполлона -15 на Луне провёл эксперимент, взял перо и молоток и одновременно их опустил с одинаковой высоты, чем подтвердил, что они падали с одинаковым ускорением. Физик Брайан Кокс в вакуумной камере НАСА провёл эксперимент и подтвердил, что перья и шар боулинга падают одновременно. Все эти опыты и эксперименты проводились, где точность измерения определялась визуально. «Для эксперимента использовались грузы – полые 300-граммовые платиново-родиевые цилиндры, вложенные в 400-граммовые полые цилиндры из титанового сплава. Кроме того, аналогичные измерения проводились еще одним идентичным акселерометром для пары цилиндров одинакового состава, платиново-родиевых. Грузы удерживались в герметичной защитной камере за счет электростатических сил и многие месяцы свободно падали, двигаясь вместе со спутником MICROSCOPE по околоземной орбите высотой более 700 км. Результат этой работы оказался тем же, что и некогда у Галилея: никакой разницы в движении между грузами не накопилось». Акселерометрами не обнаружено разницы в падении этих грузов. Точность измерения в сотни раз выше, чем визуально. Базовый принцип теории относительности выдержал новую строгую проверку — Naked Science https://naked-science.ru/article/physics/printsip-teorii-otnositelnosti-vyderzhal-proverku Сравнить точность измерения эксперимента с точностью измерения визуально? Надо признать, что эксперимент крайне спорный. Вопрос о том, что тела падают не одновременно и принцип эквивалентности нарушается для науки остаётся открытым. Эксперименты в космосе очень дорогие. Есть метод (способ) провести такой эксперимент на Земле, который с большой точностью покажет, что тела разной массы падают с разным ускорением. Все знают песочные часы и водяные часы. Вот ими и воспользуемся. Для эксперимента возьмём сжиженный газ водород и сжиженный газ радон, или сжиженный газ азот и сжиженный газ радон. Водород - атомная масса: 1,00784. Радиус атома 53 пм. Азот – атомная масса: 14,00643. Радиус атома 75 пм. Радон – атомная масса: 222,0176. Радиус атома 214 пм. Калибруем проходное отверстие в часах, например для водорода и радона. Радиус атома радона 214 пм. Радиус атома водорода 53 пм. 214 пм / 53 пм = 4,037735849056604 Калиброванное отверстие в жидкостных часах для радона должно быть больше, чем для жидкого водорода в 4, 0377 раза. Если взять в часах одинаковые отверстия, то надо с этим коэффициентом пересчитать количество проходимых атомов через сечение отверстия в единицу времени. Заливаем одинаковый объём жидкого водорода в часы для жидкого водорода и такой же объём радона в часы для жидкого радона. Так как более лёгкие тела падают быстрее тяжёлых тел, то весь жидкий водород быстрее пройдёт из верхней ёмкости часов в нижнюю часть, чем жидкий радон. По разнице времени можно рассчитать разницу ускорений их свободного падения. Этот же эксперимент можно провести на жидких металлах, например: литии и ртути. Литий – атомная масса: 6,938 Радиус атома 145 пм. Ртуть – атомная масса: 200,592. Радиус атома 157 пм. 157 пм /145 пм = 1,082758620689655 Калиброванное отверстие в жидкостных часах для ртути должно быть больше, чем для жидкого лития в 1,083 раза. С минимальными затратами в земных условиях ставиться эксперимент, в котором определяется ускорение свободного падения для лёгких и тяжёлых веществ. 28. 02. 2023 г. С уважением А.Т. Дудин.


3.04.2023, 20:38 Цорин Борис Иосифович
Отзыв: Как обычно, у этого автора, бред. "Гравитационную силу надо рассматривать, как инерционную, то есть силу сопротивления","планеты, находясь в постоянном напряжении взаимодействия, ... сопротивляются подтягиванию самих себя к Солнцу", "тела притягивают Солнце с разным ускорением, но ... эти разные ускорения передаётся самим телам" - мдааа. Еще и возмущается, что этот выверт псевдологики "игнорировался при преподавании во всех учебных заведениях и в учебниках". Отдельно поразителен вывод: автор "доказал", что чем легче планета, тем ближе к Солнцу она должна находиться. Видимо, Сатурна с Нептуном не существует, что уж говорить об астероидах...


4.04.2023, 20:32 Дудин Александр Тимофеевич
Отзыв: Уважаемый Борис Иосифович! Вы сначала разберитесь в своём бреде, а потом будите разбираться у других. Если Вам что – то не дано понять, то это не значит, что это не верно. Если Вы тяните стену резиновым жгутом и потеряли опору, то Ваше ускорение будет направлено в сторону стены. «Отдельно поразителен вывод: автор "доказал", что чем легче планета, тем ближе к Солнцу она должна находиться. Видимо, Сатурна с Нептуном не существует, что уж говорить об астероидах…» Планеты Сатурн и Нептун показывают, что они находятся под внешним гравитационным воздействием. Если рассматривать расположение спутников планет, то видим, что пояс крупных спутников находиться по «центру», от этого пояса в сторону планеты массы спутников уменьшаются, и во внешнюю сторону массы спутников уменьшаются.


5.04.2023, 19:24 Цорин Борис Иосифович
Отзыв: "Если Вы тяните стену резиновым жгутом и потеряли опору, то Ваше ускорение будет направлено в сторону стены" - о, Земля опору потеряла, караул. "Планеты Сатурн и Нептун показывают, что они находятся под внешним гравитационным воздействием" - ух они иноагенты.


5.04.2023, 19:25 Цорин Борис Иосифович
Отзыв: "Если рассматривать расположение спутников планет, то видим, что пояс крупных спутников находиться по «центру», от этого пояса в сторону планеты массы спутников уменьшаются, и во внешнюю сторону массы спутников уменьшаются" - а то, что Европа, находясь между Ганимедом и Ио, меньше их обоих по массе, несомненно, объясняется уже их гравитационным воздействием, да?


6.04.2023, 1:15 Цорин Борис Иосифович
Отзыв: Ликбез. {1} Верно ли, что ускорение свободного падения зависит от массы и меньшего тела? Да верно, в изолированной замкнутой системе. На практике же любая звезда/планета одновременно притягивается в разные стороны бесконечно большим количеством меньших или далеких тел, но влияние большинства из них не может быть замечено. {2} Верно ли, что центростремительное ускорение зависит от массы планеты? Да, верно, чем больше масса, тем больше ускорение (а не как описано в статье). {3} Влияет ли эта зависимость на расстояние от планеты до Солнца? Да, влияет, в микроскопических масштабах. {4} Открыл ли эту зависимость Дугин А.Т.? Нет, это де-факто поправка Ньютона к третьему закону Кеплера. {5} Можно ли из этой зависимости сделать вывод, что более маленькие планеты будут ближе к Солнцу? Нет, нельзя, так как радиус орбиты зависит от скорости движения планеты, а масса планеты влияет крайне мало. {6} Всегда ли соблюдается закономерность "большие спутники далеко от центрального тела, маленькие либо близко, либо очень далеко"? Нет, у других звезд есть газовые гиганты, которые ближе к своей звезде, чем Меркурий к Солнцу. Фобос больше Деймоса, а к Марсу ближе. И т.д. {7} Почему же у Солнца нет газового гиганта близко к Солнцу? Потому что нам повезло: если бы был, то нас бы тут не было. {8} Почему же у всех планет-гигантов в Солнечной системе на низких орбитах находятся исключительно маленькие спутники? Видимо, это связано с тем, как эти спутники появились. Кроме того, приливные силы увеличивают радиус орбиты, и на более крупные тела это влияет сильнее, так что если несколько спутников образовались разрушением одного, то более крупные осколки со временем уходят дальше. {9} А вообще в статье есть верные рассуждения? До слов "в данный момент подошли" идут условно верные, но малоосмысленные вычисления (условно - потому что точность записи результата превышает точность исходных данных), потом - поток бреда.


7.04.2023, 19:23 Дудин Александр Тимофеевич
Отзыв: Уважаемый, Улугбек Товфикович! Спасибо, что Вы вернулись ещё раз к этой статье. Вы рассмотрели данную статью с другой точки зрения и взяли на себя ответственность дать заключение, что: «Актуальность работы очевидна. Цели и задачи работы автором выполнены. Рекомендую к публикации в журнале Sci-article.ru.» И в этом Вы правы. А, то, что данная статья актуальна, - это факт, который подтверждается разными сайтами, даже и теми, которые Вы приводили. К сожалению, данное заключение сделано в отзыве, а не в разделе рецензий, поэтому не даёт права к публикации. А, чтобы не было сомнений, напомню, что мною приведён расчёт в отзыве от 24.12.2022 г. 21.55, для Солнечной системы расчёт приведён в статье. Кроме того в отзыве от 28.02. 2023 г. 19.10 приведён способ (метод) проверки данных утверждений. С уважением А.Т. Дудин.


8.04.2023, 2:41 Ашрапов Улугбек Товфикович
Отзыв: Уважаемый Александр Тимофеевич, желаю Вам долголетия, крепкого здоровья и дальнейшего энтузиазма! С уважением, Ашрапов У.Т.


8.04.2023, 7:00 Дудин Александр Тимофеевич
Отзыв: Уважаемый, Улугбек Товфикович! Спасибо! Взаимно! Благодарю за терпеливую продолжительную дискуссию и предоставленные материалы. С уважением А.Т. Дудин.


9.04.2023, 13:23 Цорин Борис Иосифович
Отзыв: Улугбек Товфикович, вы бы, прежде чем хвалить бред, дочитывали его дальше первых строк, а то вот порадовали Вы человека - и он сейчас с удвоенным энтузиазмом дальше бред писать будет. Автор "статьи" как раз утверждает, что перо будет быстрее падать. )


11.04.2023, 15:30 Цорин Борис Иосифович
Отзыв: Все-таки не дочитали...


11.04.2023, 20:58 Цорин Борис Иосифович
Отзыв: Видите ли, Дудин А.Т. вряд ли способен писать "более правдоподобные" статьи. А "способствовать к действию написания" в данном случае - все равно что награждать алкоголика за очередной запой.


12.04.2023, 12:55 Дудин Александр Тимофеевич
Отзыв: Уважаемые оппоненты в этой работе Вы исчерпали свой научный потенциал и не только. Я приглашаю Вас в следующую тему: «Убегающие звёзды» https://sci-article.ru/stat.php?i=1675158009 С уважением А.Т. Дудин.


12.04.2023, 13:47 Цорин Борис Иосифович
Отзыв: Вот видите, Улугбек Товфикович, Вы похвалили - и он меньше чем за неделю накропал новую статью, где ссылается на бред из этой статьи и в результате продуцирует новый бред.


3.11.2023, 13:11 Кошкин Юрий Александрович
Отзыв:  Уважаемый Александр Тимофеевич! Если взять только сугубо тему, заявленную в названии статьи "Какие тела падают быстрее лёгкие или тяжёлые?", то Ваш вывод не верен, если понимать под падением не перемещение одного подвижного тела к другому (неподвижному), а их взаимное сближение под действием сил гравитации. Тяжелые тела сблизятся быстрее, чем легкие. Например, сближение шара массой 1000000 (один миллион) кг и радиусом 100 метров с шаром массой 1000 кг и радиусом один метр произойдёт быстрее, чем если бы у второго шара была масса только один килограмм. Естественно при этом должно быть отсутствие других условий (начальной скорости, сопротивления среды и пр.). Хотя расчёты показывают, что эта разница незначительна, но она свидетельствует не в пользу Вашего вывода. Так для приведённого примера и при размещении шаров на расстоянии 1000 метров друг от друга, разность времён сближения составит около пяти сотых процента. И ещё, у Вас в статье имеется серьёзное заблуждение. Я понимаю, откуда взялась фраза "Почему притягивает большая масса меньшую массу? Потому, что одинаковые массы не притягиваются. Так как у них одинаковые ускорения, направленные в разные стороны.". Вы исходили из условия применения третьего закона И. Ньютона. Однако, для описания взаимодействия удалённых друг от друга тел этот закон совершенно не применим. Единственно подходящим для этого случая является эмпирический закон всемирного тяготения (ЗВТ), который никак не связан и не зависит от третьего закона И. Ньютона. В такой ошибке Вы не одиноки. В одной из статей данного журнала, автор, ошибочно применив этот закон, даже "логически и математически доказал" абсурдную вещь, что гравитационной константы G не должно существовать. С уважением, Юрий Александрович.


4.11.2023, 18:13 Дудин Александр Тимофеевич
Отзыв: Уважаемый Юрий Александрович! Закон всемирного тяготения И.Ньютона, через гравитационную постоянную G сводится к одной из сил третьего закона И.Ньютона. Гравитационная постоянная G является ликвидатором одной из сил третьего закона И.Ньютона. Об этом в этой работе: Тайна закона всемирного тяготения https://sci-article.ru/stat.php?i=1613732883 О том, что лёгкие тела падают быстрее, чем тяжёлые тела с одной высоты и на одно и то же массивное тело, есть расчёт в этой работе и отзыве от 24.12.2022 г. С уважением А.Т. Дудин.


4.11.2023, 18:48 Цорин Борис Иосифович
Отзыв: Дааа, тут по материалам статей и комментариев целую настоящую научную статью писать можно. На тему "Физические законы в воспаленном разуме". То тела, притягиваясь, отталкиваются, то на удаленные друг от друга тела третий закон Ньютона не действует...


7.11.2023, 9:41 Кошкин Юрий Александрович
Отзыв: Уважаемый Александр Тимофеевич! Для интереса рассчитал время сближения не абстрактных шаров из моего предыдущего комментария, а Луны и Земли. Приняв справочные значения их масс и геометрических размеров, а также нулевые начальные условия (у тел нет вращений и начальных скоростей) получил, что с расстояния 384400 км они сблизятся до соприкосновения за 521 тысячу секунд. Если масса Луны была бы в два раза больше справочной, то время сближения уменьшилось бы до 517,9 тысяч секунд, а при половинной массе Луны это время увеличилось бы до 522,6 тысяч секунд. Разница небольшая, в пределах одного процента, но она однозначно подтверждает, что при прочих равных условиях сближение тел под действием сил гравитации происходит быстрее, когда они массивнее. Отмечу, что аналитическим путём нельзя получить значение "G", также как нельзя из знания трёх законов Ньютона вывести закон всемирного тяготения (ЗВТ). Они (ЗВТ и "G") смогли быть получены только экспериментальным и наблюдательным путём. Когда для удалённых друг от друга тел уверенно записывают правильное соотношение сил, формально совпадающее с силой по третьему закону Ньютона, это совсем не означает, что для расположенных таким образом тел действует этот третий закон, как ошибочно полагают некоторые комментаторы. Эти силы смогли быть определены только из ЗВТ и не надо заблуждаться, что без знания этого закона возможно было правильно оценивать взаимодействие тел на расстоянии друг от друга. Конечно, сейчас многим кажется очевидным это соотношение сил и они не понимая, что оно определено не из третьего закона, а из ЗВТ, начинают применять третий закон вне области его действия, получая абсурдные результаты, о которых я Вам писал. И ещё. Было бы интересно сравнить результаты Ваших расчётов времени сближения Земли и Луны с моими. С уважением, Юрий Александрович.


7.11.2023, 18:33 Кошкин Юрий Александрович
Отзыв: Уважаемый Александр Тимофеевич! Кое-что захотелось добавить к предыдущему комментарию. Удивительно, но многие авторы, комментаторы и даже рецензенты не видят крамолы в применении третьего закона Ньютона к удалённым друг от друга телам. Иногда это приводит к абсурдным выводам, а иногда к бессодержательной дискуссии с ничего не значащими и не существенными аргументами спорящих. Попробую яснее показать, что взаимодействие удалённых друг от друга тел можно правильно описать только законом всемирного тяготения (ЗВТ), а третий закон Ньютона для этого не подходит, и эти законы совершенно разные и ничем между собой не связаны. Для доказательства этого примем, что нам хорошо известны все три закона Ньютона, однако закон ЗВТ мы не знаем. И вот допустим, что в какой-то нулевой момент времени мы обнаруживаем в космическом пространстве два неподвижных тела на расстоянии, например, 1000 метров. В следующий момент времени они начинают сближаться. Почему это происходит мы не понимаем, ведь ЗВТ нам неизвестен, а законы Ньютона, в том числе и третий (действие равно противодействию), ничего прояснить не могут, ведь тела, на наш взгляд, никак не взаимодействуют между собой. При сближении и ударе тел друг о друга с последующим отскоком мы немного успокаиваемся, так как вроде всё начинает с этого момента проясняться. Тела, в соответствие с третьим законом противодействовали друг другу и отскочили с ускорением, которое легко можно определить из второго закона. Но дальше начинается вновь странное, почему-то направление движения тел постепенно изменяется и они вновь начинают сближаться. Что опять же, никаким из трёх законов не может объясниться. Надеюсь из описания этого условного примера ясно видна разница между ЗВТ и третьим законом Ньютона и ограниченность область действия последнего. Ну и ошибочность применения его для описания взаимодействия удалённых друг от друга тел. Это заблуждение, к сожалению, проскакивает и в Ваших статьях. С уважением, Юрий Александрович.


7.11.2023, 19:45 Ашрапов Улугбек Товфикович
Отзыв: Уважаемый Александр Тимофеевич! Американские физики теоретически обосновали возможность существования особого типа сил, которые объясняют свойство темной материи ускользать от наблюдений. Для их описания авторы применили математический подход на основе принципа дополнительных измерений. Результаты исследования опубликованы в журнале Journal of High Energy Physics. По оценкам ученых, на темную материю приходится примерно 85 процентов материальной Вселенной. Но, в отличие от обычного вещества, темную материю нельзя ни обнаружить, ни описать ее свойства, так как она не поглощает, не отражает и не испускает свет - https://ria.ru/20210603/materiya-1735411502.html.


7.11.2023, 20:56 Дудин Александр Тимофеевич
Отзыв: Уважаемый Юрий Александрович! Вижу, что Вы хотите разобраться в этом вопросе очень серьёзно, что взялись за расчёты. Расчёта не видел, но по результатам понимаю, что Вы ускорение Земли и ускорение Луны складываете, а они образованы разными телами и приложены к разным телам, при этом направлены в разные стороны. У Вас получается, что не какой энергии, и не какой работы по перемещению одного тела другим не затрачивается. Как будь-то у Земли и Луны силы появились из вне и толкают эти два тела друг к другу. Если Земля приближает к себе Луну, то она преодолевает массу инерции Луны на всём пути, точно так же Луна, притягивая Землю, пытается преодолеть сопротивление инерции массы Земли так же на всём пути. Поэтому ускорения не складываются, а вычитаются, и чем больше будет масса Луны, тем медленнее эти два тела будут перемещаться навстречу друг другу. При меньшей массе Луны, будет меньше её сопротивление перемещению, и она быстрее будет сближаться с Землёй. Желаю Вам разобраться в этом вопросе. С уважением А.Т. Дудин.


8.11.2023, 10:00 Кошкин Юрий Александрович
Отзыв: Уважаемый Александр Тимофеевич! Сожалею, что возможно и в других новых работах, у Вас будут присутствовать фразы типа "Почему притягивает большая масса меньшую массу? Потому, что одинаковые массы не притягиваются". Но тратить время на попытки более доступней и убедительней доказать ошибочных подобных утверждений больше не буду. С уважением, Юрий Александрович.


8.11.2023, 10:45 Дудин Александр Тимофеевич
Отзыв: Уважаемый Улугбек Товфикович! Благодарю Вас за очередное посещение моей статьи и предложенный материал. Поиски тёмной материи, тёмных сил, темной энергии и других тёмных сущностей в других измерениях, это очередная целенаправленная афера затащить науку в тёмный тупик темными физиками. Вернёмся к нашим насущным проблемам. Магнитное взаимодействие, до сих пор, не изучено, как образуются магнитные поля разной конфигурации, на каком расстоянии проявляют наибольшее взаимодействие, и где образуют магнитные ямы, какие частицы участвуют в переносе магнитного взаимодействия. Не изучена роль и свойства таких частиц, как нейтрино и антинейтрино. Закон всемирного тяготения применяем односторонне, до сих пор не можем установить и измерить, что лёгкие тела падают быстрее тяжёлых тел, а это на космических просторах выливается в существенную ошибку. Гравитационные и центробежные силы применяем по отдельности, а они образуют единую систему взаимодействия, так как имеют разные зависимые законы изменения их от расстояния. Некоторые проблемы раскрыты в моих работах: Убегающие звёзды https://sci-article.ru/stat.php?i=1675158009 Гармония в угловых скоростях между планетами в Солнечной Системе https://sci-article.ru/stat.php?i=1655008723 Роль ядра Солнца и планет в перемещении тел по орбите вокруг себя https://sci-article.ru/stat.php?i=1655022445 Ускорения тел на орбите вокруг центрального тела. Часть 1 https://sci-article.ru/stat.php?i=1653847153 Ускорения тел на орбите вокруг центрального тела. Часть 2 https://sci-article.ru/stat.php?i=1653934276 Тайна закона всемирного тяготения https://sci-article.ru/stat.php?i=1613732883 Масса. - Физика - Новая Теория /электронный ресурс/ http://www.newtheory.ru/physics/massa-t6618.html С помощью простой арифметики всё просчитывается и доказывается на конкретных примерах без тёмной материи и тёмных сил на основании общеизвестных законов. С уважением А. Т. Дудин.


8.11.2023, 11:01 Дудин Александр Тимофеевич
Отзыв: Уважаемый Юрий Александрович! Вы в своих расчётах на лунах пришли к выводу, что тяжёлые тела падают быстрее, чем лёгкие. Этот результат так же противоречит общепринятому утверждению. Мои результаты Вы отмели, поэтому проблем у Вас стало меньше. Я уважаю вашу самостоятельность в поиске истины и желаю успехов на этом трудном пути. С уважением А.Т. Дудин.


8.11.2023, 14:35 Кошкин Юрий Александрович
Отзыв: Уважаемый Александр Тимофеевич! Вот я не выдержал и решил нарушить обещание самому себе прекратить обсуждать некоторые утверждения в Ваших статьях. Я не знаю, как можно доказать абсурдность утверждения "что одинаковые массы не притягиваются", если Вы не принимаете или не понимаете закон всемирного тяготения (ЗВТ)? На гораздо сложнее для понимания вопрос о том, какие тела сближаются быстрее, более массивные или менее (эта формулировка точнее, чем "Какие тела падают быстрее лёгкие или тяжёлые?" и имеет тот же смысл) проще ответить, произведя сравнительные и точные расчёты времени сближения тел с различными массами, что я и сделал. Опять же непонимание ЗВТ не убедило Вас в правильности этих расчётов и ошибочности своего вывода. Если аргументация, вытекающая из положений ЗВТ, Вас не убеждает, то других аргументов для предметного обсуждения Ваших статей у меня нет. С уважением, Юрий Александрович. P.S. На то, что в интернете есть публикации, убедительно доказывающие, что при прочих равных условиях (отсутствия сопротивления среды и пр.) тяжелые предметы достигают Землю быстрее, чем лёгкие, ссылаться не буду. При желании их нетрудно найти. Это к Вашей фразе о том, что высказывание о более быстром сближении тяжёлых тел противоречит общепринятому утверждению.


27.01.2024, 10:39 Алиханова Дилара Искандеровна
Отзыв: Прежде чем писать статьи на подобные темы автору нужно старательно изучить хороший учебник по физике (достаточно даже школьного). Может тогда в его "научных" статъях не появятся утверждения такого рода, как "Потому, что одинаковые массы не притягиваются. Так как у них одинаковые ускорения, направленные в разные стороны.".


28.01.2024, 14:35 Дудин Александр Тимофеевич
Отзыв: Уважаемая Дилара Искандеровна! Прежде чем читать статьи на подобные темы и делать отзывы «нужно старательно изучить хороший учебник по физике (достаточно даже школьного)». Вы правильно делаете, что берёте всё под сомнение, но, Вам, как будущему учёному, недопустимо огульно вешать ярлыки. Есть формула Всемирного тяготения И. Ньютона, вот с помощью её и докажите, что сделанные выводы не верны. Желаю Вам успехов. С уважением А.Т. Дудин.


28.01.2024, 19:17 Цорин Борис Иосифович
Отзыв: Экие ограничения. "Не смейте находить у меня ошибки, докажите, что я неправ, используя исключительно вот это вот".


29.01.2024, 8:24 Дудин Александр Тимофеевич
Отзыв: Уважаемый Борис Иосифович! Понимая Вашу скромность, но в этом случае надо всё – таки сказать, что все темы можно доказывать с помощью Вашей гениальной работы: «О происхождении выражения «шаром покати» в русском языке». С уважением А.Т. Дудин.


29.01.2024, 19:00 Цорин Борис Иосифович
Отзыв: Я догадываюсь, что Вы хотели сказать что-то остроумное, но, как обычно, получилось нечто бессмысленное.


29.01.2024, 19:17 Ашрапов Улугбек Товфикович
Отзыв: Учитывая два объекта одинакового размера, но изготовленных из разных материалов, более тяжелый (более плотный) объект будет падать быстрее, поскольку сила гравитации будет больше для более тяжелого объекта. Когда два объекта падают на одинаковую высоту, один тяжелее, а другой легкий и не имеет сопротивления воздуха, то они падают с одинаковой скоростью. Но если сопротивление воздуха становится слабым, то оно препятствует движению более легких объектов, поскольку более легкий объект имеет меньший вес по сравнению с более тяжелым, поэтому в этом случае более тяжелый объект падает быстрее - https://homework.study.com/explanation/does-a-heavier-object-fall-faster.html. Исходя из вышеизложенного, можно заключить, что утверждение автора "лёгкие тела падают быстрее тяжёлых тел" является ошибочным.


30.01.2024, 22:56 Дудин Александр Тимофеевич
Отзыв: Уважаемый Улугбек Товфикович! В этой работе Мы с Вами вели очень странную дискуссию, как только я обстоятельно отвечаю на Ваши вопросы, Вы удаляете свой отзыв? И вот снова, можно конечно скопировать Ваши слова отзыва, а потом отвечать на них. Ашрапов У.Т.: «Но если сопротивление воздуха становится слабым, то оно препятствует движению более легких объектов, поскольку более легкий объект имеет меньший вес по сравнению с более тяжелым, поэтому в этом случае более тяжелый объект падает быстрее - https://homework.study.com/explanation/does-a-heavier-object-fall-faster.html. Исходя из вышеизложенного, можно заключить, что утверждение автора "лёгкие тела падают быстрее тяжёлых тел" является ошибочным». Ваша ссылка, не поддаётся не какой критике, это из серии: «одна бабка сказала», и Вы на основании её, делаете свой Вывод. Улугбек Товфикович! Вы начните читать с ответа на рецензию, 12.11.2023 14:14 Ответ на рецензию автора, проанализируйте весь материал, укажите на ошибки, а потом, как учёный, делайте вывод. С уважением А.Т. Дудин.


31.01.2024, 9:28 Ашрапов Улугбек Товфикович
Отзыв: Уважаемый Александр Тимофеевич! Скорость падения чего-либо под действием силы тяжести определяется числом, известным как «ускорение силы тяжести», которое на поверхности нашей Земли составляет 9,81 м/с^2. Это означает, что за одну секунду скорость любого объекта вниз увеличится на 9,81 м/с из-за силы тяжести. Именно так работает гравитация: она ускоряет все с одинаковой скоростью. Что может смутить, так это тот факт, что сила гравитации сильнее на более тяжелых объектах, чем на более легких. В реальном мире существует сопротивление воздуха, поэтому иногда тяжелые предметы падают быстрее. Например, если вы уроните перо и бросите камень, камень приземлится первым, поскольку воздух сильнее замедляет перо. Если бы вы сделали то же самое где-нибудь, где нет воздуха (Луна), перо и камень приземлились бы точно в одно и то же время. Галилей заметил, что разные объекты падают с одинаковой скоростью, до тех пор, пока не была разработана Общая теория относительности, не было никакого объяснения, почему так происходит - https://van.physics.illinois.edu/ask/listing/164


31.01.2024, 10:23 Алиханова Дилара Искандеровна
Отзыв: Удивительно, что статья автора, не понимающего простого закона всемирного тяготения и утверждающего что тела одинаковой массы не притягиваются друг к другу, вызвала дискуссию. Вывод в статье ошибочен, тяжелое тело упадёт (сблизится) быстрее, чем легкое. Проведём расчёт. Отпустим с высоты 1100 метров шары радиусом 100 метров с массой 1 000 000 000 и 10 000 000 000 кг. Если не учитывать сопротивление воздуха, то второй шар коснётся поверхности Земли быстрее первого на 1,07 на десять минус в четырнадцатой степени секунд. Разница небольшая, так как массы шаров намного меньше массы Земли, но она надежно свидетельствует об ошибочности вывода в статье. Поясню специально для автора, что шары не только падают на Землю, но и Земля падает на шары. Поэтому расстояние до сближения шаров с Землёй составляет не 1000 метров (высота 1100-радиус 100), а меньше. И чем массивнее тела, тем расстояние до сближения будет больше укорачиваться. Это и вызывает разницу во времени падения (сближения), хотя и незначительную из-за малости масс шаров, рассмотренных в примере. Подробнее об этом сказано в хороших школьных учебниках, прочесть которые я автору и советую.


31.01.2024, 15:40 Дудин Александр Тимофеевич
Отзыв: Уважаемая Дилара Искандеровна! Из Вас получиться хороший учёный. Вы взялись за расчёты и рассуждения, а это первый и верный шаг к успеху. Но таких шагов придётся сделать не одну тысячу. И не спешите делать выводы. В своих расчётах Вы взяли шары больших масс, но они отличаются друг от друга, только на один порядок. А можно взять для расчёта массу 1000 кг и массу 0,01кг, и они будут отличаться на 5 порядков. Гравитация,- это взаимодействие масс. А что такое масса? Вы считаете ускорение сближения тел, складывая ускорения от каждого тела в отдельности, и получаете, что более тяжёлое тело падает быстрее, чем более лёгкое. А где инерция масс? Вы правильно пишите, что шары оказывают взаимодействие на Землю. Для того чтобы оказывать взаимодействие на Землю, надо «упираться», и в этом случае большая масса больше «упирается». Чтобы переместить большую массу, из одной точки пространства в другую, надо затратить больше энергии, чем переместить меньшую массу из этой точки в ту же, что и большую массу. Гравитация, - это преодоление инерции масс. Если Вас действительно интересует эта тема, то Вы не спеша разберитесь со всеми доводами и расчётами. Один отзыв по объёму просто не позволяет раскрывать всю тему. Да и Вам надо поработать. С уважением А.Т. Дудин.


31.01.2024, 16:24 Дудин Александр Тимофеевич
Отзыв: Уважаемый Улугбек Товфикович! Спасибо, что уделяете большое внимание этой работе в отзывах, но содержание самой работы Вы, наверное, забыли? «Скорость падения чего-либо под действием силы тяжести определяется числом, известным как «ускорение силы тяжести», которое на поверхности нашей Земли составляет 9,81 м/с^2. Это означает, что за одну секунду скорость любого объекта вниз увеличится на 9,81 м/с из-за силы тяжести. Именно так работает гравитация: она ускоряет все с одинаковой скоростью» У.Т.Ашрапов. И это верно, можно взять космическое тело, например, массой равной Луне и с одного и того же расстояния она будет одинаково влиять, что на Меркурий, что на Венеру, что на Юпитер, пытаться обеспечить им одинаковые ускорения, но только реакции этих космических тел будут разными. Это Вы можете проверить расчётами, подобные расчёты приведены в статье. «Галилей заметил, что разные объекты падают с одинаковой скоростью, до тех пор, пока не была разработана Общая теория относительности, не было никакого объяснения, почему так происходит - https://van.physics.illinois.edu/ask/listing/164» У.Т.Ашрапов. Уважаемый Улугбек Товфикович! Ссылка на этот сайт отличается от предыдущего, тем, что в этом сайте: «Продолжение № 2: тяжелый падает быстрее? Один парень сказал мне, что при …». Авторы Аноним, Тамара, общеизвестные авторитеты, и это неоспоримо, и очень научно! А про А. Эйнштейна, скажу Вам по секрету, что он перепутал магнитные поля космических тел с искривлённым пространством. С уважением А.Т. Дудин.


31.01.2024, 18:40 Алиханова Дилара Искандеровна
Отзыв: Уважаемый Александр Тимофеевич! Своим ответом Вы лишний раз доказали, что совсем не понимаете закон всемирного тяготения. Я Вам привела результаты расчёта примера с думя шарами разных масс. Проверьте эти результаты, если сомневаетесь в моём выводе. Зачем пустые слова о том, что можно взять для расчёта массу 1000 кг и массу 0,01кг или отвлечённые рассуждения о том "чтобы переместить большую массу, из одной точки пространства в другую, надо затратить больше энергии, чем переместить меньшую массу из этой точки в ту же, что и большую массу". Я не сомневаюсь, что можно для расчёта взять тысячи других масс и что для перемещения более массивного тела нужно совершить больше работы, но какое отношение это имеет к численной проверке того, какие тела падают (сближаются) быстрее? Просто на цифрах подтвердите или опровергните мои выводы, конкретно и не виляя. Если хотите, возьмите другие массы, отличающиеся на сколь угодно порядков. Возможно всё внимательно и точно посчитав, Вы поймёте ошибочность Вашего понимания ЗВТ.


31.01.2024, 22:00 Дудин Александр Тимофеевич
Отзыв: Уважаемая Дилара Искандеровна! Извините, но хамству, Вы учитесь быстрее, чем науке. А эти два направления несовместимы. Вам дали решить задачу, и Вы начинаете нервничать? Где ваше решение, чтобы искать ошибки, или Вы хотите, чтобы я решил за Вас? Читайте статью, там есть вся методика решений. Выводы Ваши недопустимы, если Вы настаиваете на них, то Вам надо искать других авторов, на этом сайте они есть. Гранит науки надо грызть в одиночку. Желаю успеха. С уважением А.Т. Дудин.


1.02.2024, 6:58 Цорин Борис Иосифович
Отзыв: {1} Вот мне интересно, когда гражданин Дудин критикует за ссылки на кого попало, он осознает, что сам в своих работах постоянно точно так же ссылается на кого попало (то есть сам на себя)? {2} Аргумент "Вот тут на первых попавшихся числах посчитано - так мне числа не понравились, возьмите другие числа" великолепен. {3} "Чтобы переместить большую массу, из одной точки пространства в другую, надо затратить больше энергии, чем переместить меньшую массу из этой точки в ту же, что и большую массу" - ага, то есть, я гляжу, старые бредовые доводы про "взаимодействие в обратную сторону" и "невзаимодействие тел равной массы" больше не приводятся, приводятся новые. Ну что ж... Во-первых, само утверждение ошибочно, на перемещение энергия не расходуется, она вообще ни на что не расходуется, а только переходит из одной формы в другую, называется "закон сохранения энергии". Во-вторых, при перемещении энергия и из одной формы в другую не переходит, кинетическая энергия повышается при увеличении скорости. В-третьих, если тело именно падает, а не движется по орбите, то кинетическая энергия повышается за счет уменьшения гравитационной, которая точно так же пропорциональна массе тела, как и кинетическая, так что Ваш довод снова противоречит науке. {3a} Впрочем, все, что касается науки, в спорах с гражданином Дудиным, проще оставлять в стороне. Так что вот довод, который ему должен быть понятен. "Затратить больше энергии" надо тому, кто грузы поднимал в ту точку, откуда они падали. Так что проводите смело эксперимент: возьмите легкий мячик одной рукой, тяжелую гирю другой рукой, и одновременно бросьте вверх. Вот увидите (ну или если метко гирю бросите, уже не увидите): гиря на голову упадет куда быстрее мячика. Так что в рамках довода "затратить больше энергии" тяжелое тело упадет быстрее легкого.


1.02.2024, 8:37 Алиханова Дилара Искандеровна
Отзыв: Александр Тимофеевич, если не хотите адекватно воспринимать замечания и аргументированно на них отвечать, то это конечно Ваше право. Также Вы конечно вправе писать статьи на любые темы, даже в которых Вы не разбираетесь и не хотите (или не способны) разобраться. Пытаться объяснить Вам абсурдность высказывания о том, что "тела одинаковой массы не притягиваются друг к другу" и понимания основ ЗВТ в целом смысла больше не вижу. Если о статье, то её научной назвать нельзя.


1.02.2024, 17:55 Ашрапов Улугбек Товфикович
Отзыв: Из опыта и знания: - Перья падают медленнее, чем более тяжелые предметы.- Парашютисты падают медленнее, когда парашют раскрыт.- Тяжелые предметы давят на вашу руку с большей силой.- Вы можете почувствовать силу на своей руке, если высунете ее из окна движущейся машины. Вопрос: 1) Что падает быстрее в свободном падении: перо или молот? Ответ: Оба объекта падают с одинаковой скоростью. 2) Изменяет ли масса ускорение объекта, если единственной силой, действующей на него, является гравитация? Ответ: Масса не влияет на скорость падающих объектов, если предположить, что на нее действует только гравитация. 3) Что заставляет перо падать на землю медленнее, чем молоток? Ответ: Сопротивление воздуха замедляет падение пера. 4) Почему тяжелые предметы падают быстрее? Ответ: Тяжелый предмет всегда падает быстрее легкого, если его сбросить с высоты. Причина: сила гравитации пропорциональна массе объекта.


2.02.2024, 11:11 Алиханова Дилара Искандеровна
Отзыв: Уважаемый Улугбек Товфикович! Осмелюсь обратить внимание на полное противоречие между Вашими высказываниями "Изменяет ли масса ускорение объекта, если единственной силой, действующей на него, является гравитация? Ответ: Масса не влияет на скорость падающих объектов, если предположить, что на нее действует только гравитация" и "Тяжелый предмет (надо полагать более массивный, это уже моё примечание) всегда падает быстрее легкого, если его сбросить с высоты". Однако Ваш вывод, что тяжелые тела сближаются (падают) быстрее, чем лёгкие, конечно правильный, но это происходит только из-за того, одно из взаимодействующих тел, например Земля, "вытягивает" на себя падающий предмет, а предмет в свою очередь "вытягивает" на себя Землю. При этом расстояние, которое они проходят до соприкосновения уменьшается по сравнению с начальным расстоянием между телами и это разность будет тем значительней, чем больше тела будут "вытягивать" себя друг к другу, то есть чем массивнее (тяжелее) они будут. Возможно это не выглядит столь однозначным для всех. Например, автор считает, что увеличение массы падающего тела увеличивает инерцию, что приводит к увеличению времени падения. Поэтому желательно это проверить расчётом. Для этого я рассмотрела следующий пример, мысленно опуская на Землю с высоты 1100 метров шары радиусом 100 метров, массой 1 000 000 000 кг и 10 000 000 000 кг при начальной нулевой скорости и отсутствия сопротивления среды. В связи с тем, массы шаров многократно меньше массы Земли, расчёт необходимо было проводить с большой точностью, поэтому заранее прошу извинения о том, что привожу громоздкие числа. Время падения шара массой 10 000 000 000 кг оказалось меньше на 1.065814103640150278806686401 на десять в минус 14 степени секунд меньше, чем шара массой 1 000 000 000 кг. Я предложила проверить этот расчёт автору, но похоже он не способен с такой точностью этого сделать. Вместо того, чтобы в этом признаться, он занервничал и почему-то обвинил меня в хамстве, то есть повёл себя как уличенный в чем-то подросток. Для интереса провела расчёты и с другими массами шаров (100 000 000 000 и более.), вывод везде однозначен - массивные тела сближаются быстрее легких.


2.02.2024, 16:00 Дудин Александр Тимофеевич
Отзыв: Уважаемый Улугбек Товфикович! Кроме расчётов, приведённых в статье, доказывающих, что лёгкие тела, падая свободно в вакууме, имеют большее ускорение падения, чем тяжёлые тела, этот факт и доказывать не надо, он доказан И. Ньютонам в третьем законе: Ma(1) = ma(2). «Вопрос: 1) Что падает быстрее в свободном падении: перо или молот? Ответ: Оба объекта падают с одинаковой скоростью. 2) Изменяет ли масса ускорение объекта, если единственной силой, действующей на него, является гравитация? Ответ: Масса не влияет на скорость падающих объектов, если предположить, что на нее действует только гравитация. … Ответ: Тяжелый предмет всегда падает быстрее легкого, если его сбросить с высоты. Причина: сила гравитации пропорциональна массе объекта» У.Т.Ашрапов. Падение пера и наполненного свинцом шара в вакууме будет происходить с одинаковой... - поиск Яндекса по видео https://yandex.ru/video/preview/4509983386356630396 Посмотрите внимательно в самом начале съёмки, перья выше шара см около 5 и перед их касанием ящика с песком, где они вровень с шаром. С уважением А.Т. Дудин.


4.02.2024, 12:01 Ашрапов Улугбек Товфикович
Отзыв: Уважаемый Александр Тимофеевич, и конечный результат эксперимента в видео ролике (https://yandex.ru/video/preview/4509983386356630396) и слова комментарий под ним подтверждают - "Если бросить два предмета в вакууме (а не в воздухе), то они будут падать с одинаковой скоростью!". Парашютисты в воздухе падают медленнее, когда парашют раскрыт (парашют служит для замедления скорости движения предмета в воздухе). Сила гравитации пропорциональна массе объекта и подтверждение этого может быть изучение метеоритов. На месте падения крупного метеорита при высоких скоростях (2000-4000 м/с) может образоваться большой кратер, например Аризонский кратер (диаметр 1200 м), а при невысоких скоростях (несколько сотен м/с) диаметр образующегося ударного кратера сравним с размерами самого метеорита, и даже крупные метеориты могут хорошо сохраниться, как например метеорит Гоба (60 тонн).


12.02.2024, 21:32 Дудин Александр Тимофеевич
Отзыв: Уважаемый Улугбек Товфикович! Этому опыту более 9 лет и в интернете можно найти оригиналы съёмки очень высокого качества. А ещё на видео есть настройки, позволяющие значительно увеличивать скорость просмотра и замедлять её, и кнопкой стоп останавливать в любое мгновение. Я многократно просмотрел эти ролики и убедился, что перья падают быстрее (проседают) относительно шара. Шары для боулинга имеют размеры: 21,59 – 21,83 см (7,3 кг.) То, что перья падают быстрее, нет сомнений. Скорость падения лёгких тел подтверждается расчётами. А в интернете нашёл несколько работ, которые так же, при проведении опытов, фиксируют это явление. https://fok-delfin.ru/period-kolebanii-matematiceskogo-mayatnika-zavisimost-ot-massy-ili-neobxodimost-uceta-drugix-faktorov Изменение периода колебаний от амплитуды было установлено Х. Гюйгенсом, зависимость периода колебания от массы маятника столь же очевидна. Все наблюдали и, вероятно, участвовали в аттракционе карусели на цепочках, где наглядно видно, как карусель при набирании оборотов с лёгким весом быстрее, чем с тяжёлым весом поднимается вверх, и при снижении оборотов, быстрее опускается вниз. Примерно подобные процессы происходят с маятниками разной массы при каждом периоде колебаний. Более массивная масса маятника имеет большую инерцию и на большее время «зависает» в нулевой точке при смене направления движения. Есть фокус с выдёргиванием салфетки на столе из под подноса с разной посудой, где легче выдернуть салфетку из под большей массы, так как инерция «покоя» дольше сохраняет эту массу в первоначальном состоянии. Есть пословица: «Шило в мешке не утаишь». Меня удивляет тот факт, что есть расчёты, но они во внимание не принимаются. С уважением А.Т. Дудин.


14.02.2024, 7:45 Цорин Борис Иосифович
Отзыв: {1} "Посмотрите внимательно в самом начале съёмки, перья выше шара см около 5 и перед их касанием ящика с песком, где они вровень с шаром" - неверно. Шар находится выше середины перьев, но ниже верхушки перьев в начале падения и шар находится выше середины перьев, но ниже верхушки перьев в конце падения. Более точно сравнить нельзя из-за разных ракурсов съемки. {2} "Более массивная масса маятника имеет большую инерцию и на большее время «зависает» в нулевой точке при смене направления движения" - неверно. Зависание в нулевой точке при смене направления движения длится ровно 0 секунд, остальное время, которое кажется "зависанием" - медленное движение. "Большая инерция" означает то, что для вдвое большей массы надо приложить вдвое большую силу. Но тяготение как раз и действует с вдвое большей силой на вдвое большую массу, так что инерцию Вы зря приплетаете. Впрочем, как и всё остальное.


14.02.2024, 7:50 Цорин Борис Иосифович
Отзыв: {3} "Меня удивляет тот факт, что есть расчёты, но они во внимание не принимаются" - Вы до сих пор не осознали, что Ваши расчёты в этой статье сводятся к "Взяли ПЯТЬ значащих цифр, покрутили-покрутили, получили разницу в ШЕСТОЙ значащей цифре там, где по теории должны быть равные значения". Я Вам такими расчетами что угодно опровергну. Ну например... Встаю на весы. Они показывают 78.2 кг. Беру мерный стакан, наливаю 50 мл воды. Выпиваю. Встаю на весы. Они показывают 78.3 кг. Делаем вывод: 50 мл воды весят 0.1 кг, значит, литр воды весит 2 кг, все справочники врут, я совершил великое открытие. Вот примерно такие расчеты и у Вас.


14.02.2024, 14:56 Ашрапов Улугбек Товфикович
Отзыв: Уважаемый Александр Тимофеевич, очень четко и конкретно написал Цорин Борис Иосифович в отзыве от 14.02.2024г. и я полностью согласен с ним.


16.02.2024, 10:53 Дудин Александр Тимофеевич
Отзыв: Уважаемый Борис Иосифович! «Вы до сих пор не осознали, что Ваши расчёты в этой статье сводятся к "Взяли ПЯТЬ значащих цифр, покрутили-покрутили, получили разницу в ШЕСТОЙ значащей цифре там, где по теории должны быть равные значения", удивляюсь, что Вы, как преподаватель математики могли такое сказать? То, что гравитационные воздействия слабее других сил, это не значит, что их надо игнорировать. Не надо сводить гравитационные взаимодействия к одностороннему взаимодействию. Возможно, есть взаимодействия намного слабее, чем гравитационные, Вы их запрещаете искать. А вдруг у них ускорения окажутся меньше, чем Вам хотелось бы. Вы позицию инквизитора не занимайте, а прежде, чем отзывы писать разберитесь в статье и отзывах к ней. Эксперимент без расчёта это не эксперимент. А утверждать об одинаковом ускорении падающих тел с разной массой, это отрицать взаимодействие тел. Это равносильно утверждать, что нуклоны большего тела притягивают нуклоны меньшего тела, при этом нуклоны меньшего тела на большее тело не действуют. Для расчёта возьмём пробное тело m(1) = 1 гр = 0,001 кг = 10^-3 кг и пробное тело m(2) = 10 кг Масса Земли: M = 5,9726 *10^24 кг Средний радиус Земли: r = 6371,0 км Высота сбрасывания груза над Землёй = 10 м R = r + h = 6371000 м + 10 м = 6371010 м = 6,37101 * 10^6 м G = 6,67430 *10^-11 м^3 / кг с^2 По формуле тяготения И. Ньютона определим ускорения пробных тел Землёй. F(1) = G Mm(1) / R^2 F(1) = m(1) g(З) = G Mm(1) / R^2 g(З) = G M / R^2 g(З) = 6,67430 *10^-11 м^3 / кг с^2 * 5,9726 *10^24 кг / (6,37101 * 10^6 м)^2 = 0,9820929197580721 *10 м/с^2 g(З) = 9,820929197580721 м/с^2 С таким ускорением Земля притягивает пробное тело m(1) Находим ускорение, с которым Земля воздействует на пробное тело m(2) F(2) = G Mm(2) / R^2 F(2) = m(2) g(З) = G Mm(2) / R^2 g(З) = G M / R^2 g(З) = 6,67430 *10^-11 м^3 / кг с^2 * 5,9726 *10^24 кг / (6,37101 * 10^6 м)^2 = 0,9820929197580721 *10 м/с^2 g(З) = 9,820929197580721 м/с^2 Как видим, Земля не зависимо от массы любому пробному телу сообщает одинаковое ускорение: g(З) = 9,820929197580721 м/с^2 Находим, каким ускорением воздействует на Землю пробное тело m(1). F = G Mm(1) / R^2 F = M g(1) = G Mm(1) / R^2 g(1) = G m(1) / R^2 g(1) = 6,67430 *10^-11 м^3 / кг с^2 * 10^-3 кг / (6,37101 * 10^6 м)^2 = 1,644330642865874* 10^-26 м/с^2 g(1) = 1,644330642865874* 10^-26 м/с^2 Находим, каким ускорением воздействует на Землю пробное тело m(2). F = G Mm(2) / R^2 F = M g(2) = G Mm(2) / R^2 g(2) = G m(2) / R^2 g(2) = 6,67430 *10^-11 м^3 / кг с^2 * 10 кг / (6,37101 * 10^6 м)^2 = 1,644330642865874* 10^-22 м/с^2 g(2) = 1,644330642865874* 10^-22 м/с^2 g(1) = 1,644330642865874* 10^-26 м/с^2 Как видим, разница в ускорениях на 4 порядка, эти ускорения очень малы и на Земле они значения не имеют ими можно пренебрегать, но для космоса этот результат имеет огромное значение. Результирующее ускорение для лёгкого тела будет больше, чем для тяжёлого тела: g(З) - g(1) > g(З) - g(2) Лёгкие тела и на Земле падают быстрее тяжёлых тел, но этот факт не доступен для измерения. С уважением А.Т. Дудин.


16.02.2024, 15:50 Ашрапов Улугбек Товфикович
Отзыв: В 1971 году астронавт Дэвид Скотт провел на Луне знаменитый эксперимент Галилея с падением молота и пера во время миссии «Аполлон-15». Галилей пришел к выводу, что все объекты, независимо от массы, падают с одинаковой скоростью, однако сопротивление воздуха (как в случае с пером в атмосфере Земли) может заставить перо падать медленнее. Ну, на Луне нет атмосферы (вакуума), поэтому объекты должны падать с одинаковой скоростью. Диспетчер миссии Джо Аллен в предварительном научном отчете «Аполлона-15» писал: "На последних минутах третьего выхода в открытый космос был проведен короткий демонстрационный эксперимент. Тяжелый предмет (алюминиевый геологический молоток массой 1,32 кг) и легкий предмет (соколиное перо массой 0,03 кг) были выпущены одновременно примерно с одинаковой высоты (около 1,6 м) и упали на поверхность. В пределах точности одновременного выброса было замечено, что объекты претерпели одинаковое ускорение и одновременно ударились о лунную поверхность, что было результатом, предсказанным хорошо зарекомендовавшей себя теорией, но, тем не менее, результат обнадеживающий, учитывая как количество зрителей, ставших свидетелями эксперимента. и тот факт, что путь домой был критически основан на обоснованности конкретной проверяемой теории. Джо Аллен, НАСА SP-289, Предварительный научный отчет Аполлона-15, Краткое изложение научных результатов, стр. 2-11" - https://www.mentalfloss.com/article/22913/hammer-and-feather-drop-moon.


16.02.2024, 19:59 Цорин Борис Иосифович
Отзыв: {1} "Удивляюсь, что Вы, как преподаватель математики могли такое сказать? То, что гравитационные воздействия слабее других сил, это не значит, что их надо игнорировать" - Вы, как обычно, ничего не поняли, но много наговорили. Перечитайте еще раз пример про 50 мл воды. Изучите хотя бы по школьным учебникам физики, что такое погрешность и как она влияет на результат вычислений. {2} "но этот факт не доступен для измерения" - о, Вы наконец-то это осознали, теперь Вы удалите свои статьи про маятники? {3} "g(З) - g(1) > g(З) - g(2)" - Вы в очередной раз повторили свою ошибку. Ускорения направлены навстречу друг другу, значит, их надо складывать, а не вычитать.


17.02.2024, 7:29 Дудин Александр Тимофеевич
Отзыв: "В 1971 году астронавт Дэвид Скотт провел на Луне знаменитый эксперимент Галилея с падением молота и пера во время миссии «Аполлон-15»". Уважаемый Улугбек Товфикович! Опыт, если он был, несомненно, заслуживает внимания. Но визуально определить его результаты невозможно. Перо находиться в левой руке, молоток находиться в правой руке, «одновременно» отпустить, это примерно разница в 0,01 с. Масса пера 0, 03 кг, обозначим m(1) Масса молотка 1,32 кг, m(2) Масса Луны: M = 7,3477*10^22 кг Радиус Луны: 1738,14 км Ускорение свободного падения 1,62 м/с^2 Перо и молоток падали 1,3 с. Высота падения 1,37 м R = r + h = 1738140 м + 1,37 м = 1738141,37 м G = 6,67430 *10^-11 м^3 / кг с^2 По формуле тяготения И. Ньютона определим ускорение пера Луной. F(1) = G Mm(1) / R^2 F(1) = m(1) g(Л) = G Mm(1) / R^2 g(Л) = G M / R^2 g(Л) = 6,67430 *10^-11 м^3 / кг с^2 *7,3477 *10^22 кг / (1,73814137 * 10^6 м)^2 = 1,623255738593231 м/с^2 g(Л) = 1,623255738593231 м/с^2 С таким ускорением Луна притягивает перо. По формуле тяготения И. Ньютона определим ускорение молотка Луной. F(2) = G Mm(2) / R^2 F(2) = m(2) g(Л) = G Mm(2) / R^2 g(Л) = G M / R^2 g(Л) = 6,67430 *10^-11 м^3 / кг с^2 *7,3477 *10^22 кг / (1,73814137 * 10^6 м)^2 = 1,623255738593231 м/с^2 g(Л) = 1,623255738593231 м/с^2 С таким ускорением Луна притягивает молоток. Находим, каким ускорением воздействует на Луну перо m(1). F = G Mm(1) / R^2 F = M g(1) = G Mm(1) / R^2 g(1) = G m(1) / R^2 g(1) = 6,67430 *10^-11 м^3 / кг с^2 * 0,03 кг / (1,73814137 * 10^6 м)^2 = 0,1151971890525798 *10^-23 м/с^2 g(1) = 0,1151971890525798 *10^-23 м/с^2 Находим ускорение пера: g(П) = g(Л) - g(1) = 1,623255738593231 м/с^2 - 0,1151971890525798 *10^-23 м/с^2 Находим, каким ускорением воздействует на Луну молоток m(2). F = G Mm(2) / R^2 F = M g(2) = G Mm(2) / R^2 g(2) = G m(2) / R^2 g(2) = 6,67430 *10^-11 м^3 / кг с^2 * 1,32 кг / (1,73814137 * 10^6 м)^2 = 2,916147331740634*10^-23 м/с^2 g(2) = 2,916147331740634*10^-23 м/с^2 Находим ускорение молотка: g(М) = g(Л) - g(2) = 1,623255738593231 м/с^2 - 2,916147331740634*10^-23 м/с^2 g(П) > g(М) Ускорение пера больше, чем ускорение молотка на: 2,916147331740634*10^-23 м/с^2 - 0,1151971890525798 *10^-23 м/с^2 = 2,800950142688054*10^-23 м/с^2 Зафиксировать разницу ускорений этих предметов на Луне, даже с помощью лазерных приборов проблематично. Для космоса, где разница в массах очень большая, а время исчисляется в миллиардах лет, огромное значение имеет сам факт, что более лёгкие тела падают быстрее, чем более тяжёлые тела. С уважением А.Т. Дудин.


17.02.2024, 14:36 Дудин Александр Тимофеевич
Отзыв: Уважаемый Борис Иосифович! «Вы, как обычно, ничего не поняли, но много наговорили. Перечитайте еще раз пример про 50 мл воды. Изучите хотя бы по школьным учебникам физики, что такое погрешность и как она влияет на результат вычислений». Меня Ваше пустословие только удивляет, но не вдохновляет, и не даёт не какой полезной информации. Про воду Вы сами себе поверили. Вы в расчётах разберитесь, и укажите, какая погрешность, и на какой результат могла повлиять. Расчёт проведён по совершенно одинаковым данным, и одинаковым формулам, разница только в массах пробных тел. Какие погрешности, и на что могут влиять в данном случае? «"но этот факт не доступен для измерения" - о, Вы наконец-то это осознали, теперь Вы удалите свои статьи про маятники?»,- Цорин Б.И. Вам рекомендую прочитать статью, так как Вы до сих пор этого не можете осознать, маятник,- это универсальный датчик для измерения ускорения свободного падения, а период маятника зависит не только от длины маятника, но и от амплитуды колебаний и массы маятника. За один период маятник дважды падает и дважды поднимается, то - есть пройденная высота увеличивается в 4 раза, при этом проявляются, как гравитационная сила, так и инерционная. Рекомендую Вам сходить покачаться на качелях, там Вы отдельно почувствуете, и свободное падение, и инерцию взлёта, а заодно, и переход из одного состояния в другое, и реальное время перехода, и можно сравнить периоды колебаний от массы. К тому же, приобретёте много положительных эмоций, продлите себе жизнь, и отзывы не будут столь мрачными и безысходными. «"g(З) - g(1) > g(З) - g(2)" - Вы в очередной раз повторили свою ошибку. Ускорения направлены навстречу друг другу, значит, их надо складывать, а не вычитать». Ускорения надо складывать, когда тела независимо друг от друга перемещаются на встречу друг другу. В данном случае, каждое тело перемещается за счёт другого тела. Масса тела определяется средой, в которой это тело находиться. Если Вас заставить тянуть наковальню в 100 кг по земле, будет очень проблематично, если наковальня будет в воде, тянуть станет легче, но не намного, а если её отпустить в ртуть, то она будет плавать, и Вы её перетяните без особых затрат. Для масс в космосе средой является вакуум. Чтобы одна масса могла тянуть к себе другую массу, ей надо «упираться» в среду. Это аналогично перетягивать каната, рекомендую, помогает осознать многие процессы по физике. Желаю Вам успеха. С уважением А.Т. Дудин.


18.02.2024, 14:41 Цорин Борис Иосифович
Отзыв: {1} "Расчёт проведён по совершенно одинаковым данным, и одинаковым формулам, разница только в массах пробных тел" - могу повторить другими словами, раз с первой попытки у Вас прочитать не вышло. Массы "пробных тел" - Венеры и Земли - Вы взяли до пятой значащей цифры. Следовательно, на любом этапе вычислений надо результат учитывать с аналогичной погрешностью, а при умножении и делении относительная погрешность еще и складывается. Когда я вижу, что Вы получаете полтора десятка значащих цифр из пяти, мне становится смешно. {2}"то - есть пройденная высота увеличивается в 4 раза" - ага, то есть 1,644330642865874* 10^-26 мы не можем измерить, но стоит увеличить в 4 раза - и разница получается огромная, да? {3} "Ускорения надо складывать, когда тела независимо друг от друга перемещаются на встречу друг другу. В данном случае, каждое тело перемещается за счёт другого тела" - полный бред, и вполне возможно, даже в медицинском понимании этого слова. Если паровозы навстречу друг другу будут тянуть огромные магниты, они не столкнутся из-за одинаковых ускорений, по-Вашему? Проверьте на игрушечных. {4} "Чтобы одна масса могла тянуть к себе другую массу, ей надо «упираться» в среду" - аналогично полный бред. Если я буду подтягивать к себе по канату нечто в отсутствии среды, в вакууме, у меня будет получаться. И если я буду подтягивать нечто легкое, то получаться будет легко. А если наковальню в сто кило, то в вакууме и невесомости я ее таки сдвину, просто при помощи трехметрового каната я сдвину ее меньше, чем на полтора метра, и сам сдвинусь на остальную длину каната. Третий закон Ньютона. {5} То, что при расчете движения Земли по орбите вокруг Солнца надо учитывать и массу Земли, открыл еще Ньютон (поправка Ньютона к закону Кеплера). Естественно, учитывать со знаком "плюс", а не как Вы бредите. Вы хотите утверждать, что поправка Ньютона неверна, а астрономы за сотни лет не обнаружили, что с поправкой Ньютона формулы перестают соответствовать наблюдениям, и только Вы без всяких наблюдений это поняли? )


19.02.2024, 21:43 Дудин Александр Тимофеевич
Отзыв: Уважаемый Борис Иосифович! « ..Массы "пробных тел" - Венеры и Земли - Вы взяли до пятой значащей цифры. Следовательно, на любом этапе вычислений надо результат учитывать с аналогичной погрешностью, а при умножении и делении относительная погрешность еще и складывается…». Округляя на каждом этапе вычислений, Вы внесёте такое количество погрешностей, что после этих ваших опусов, возникает сомнение, что Вы действительно умеете считать на калькуляторе? «…В данном случае, каждое тело перемещается за счёт другого тела" - полный бред, и вполне возможно, даже в медицинском понимании этого слова». Скажите, Вы умный или прикидываетесь? Отвечать не надо. Ответ, Вы, очень подробно изложили в отзывах на мои работы. «…То, что при расчете движения Земли по орбите вокруг Солнца надо учитывать и массу Земли, открыл еще Ньютон (поправка Ньютона к закону Кеплера). Естественно, учитывать со знаком "плюс", а не как Вы бредите. Вы хотите утверждать, что поправка Ньютона неверна, а астрономы за сотни лет не обнаружили, что с поправкой Ньютона формулы перестают соответствовать наблюдениям, и только Вы без всяких наблюдений это поняли? )», - Цорин Б.И. Полное не знание и не понимание законов Кеплера и законов Ньютона. Т(1)^2(M + m(1)) / Т(2)^2(M + m(2)) = a(1)^3 / a(2)^3 Здесь И. Ньютон внёс поправку к периоду, он показал, что период зависит от массы планеты. Чем больше масса планеты, тем больше период, аналогично период маятника зависит от массы, чем больше масса, тем больше период. С уважением А.Т. Дудин.


20.02.2024, 6:59 Цорин Борис Иосифович
Отзыв: {1} "Округляя на каждом этапе вычислений, Вы внесёте такое количество погрешностей" - неправильная трактовка моих слов. Для дальнейших вычислений промежуточный результат округлять не надо, в последовательности вычислений округление не участвует. Зато при вычислениях надо постоянно считать накопленную погрешность. Подсказываю (раз уж в учебник Вы лезть отказываетесь): при сложении и вычитании величин складываете абсолютную погрешность, при умножении и делении складываете относительную погрешность, при делении на число с большой относительной погрешностью производите деление на "число+погрешность" и на "число-погрешность", изучаете разницу результатов. И вот когда Вы какое-то число из своих вычислений пишете в статье, надо как минимум писать его в округленном вида, выбирая округление по размеру погрешности, а лучше бы и с указанием погрешности. В этой статье Ваши вычисления до кучи знаков после запятой просто вызывают смех и делают расчеты бессмысленными, а ошибки возникают по другим причинам, а вот ранее Вы публиковали статьи, где из подобных ошибок в расчетах делали далекоидущие выводы. {2} "Здесь И. Ньютон внёс поправку к периоду, он показал, что период зависит от массы планеты" - нагуглить Вы сумели, понять нет. Изучите не формулировку закона, а причину внесения этой поправки. Массы складываются, потому что силы тяготения складываются. А Вы эти силы вычитать пытаетесь вместе с ускорениями. {3} "Чем больше масса планеты, тем больше период" - еще и математику не знаете. При фиксированных полуосях орбит чем больше масса, тем МЕНЬШЕ период. Квадрат периода обратно пропорционален сумме массы планеты и Солнца (или иных взаимодействующих тел). T^2=4*(Pi^2)*(a^3)/(G*(M+m)).


20.02.2024, 16:05 Ашрапов Улугбек Товфикович
Отзыв: Уважаемый Александр Тимофеевич, 1) Все предметы падают на землю с постоянным ускорением, называемым ускорением силы тяжести (при отсутствии сопротивления воздуха). Она постоянна и не зависит от массы объекта. Следовательно, тяжелый предмет не падает быстрее, чем легкий и наоборот легкий предмет не падает быстрее, чем тяжелый [https://www.doubtnut.com/qna/385066904]; 2) Галилей сделал вывод, что все тела падают с одинаковым ускорением (если пренебречь сопротивлением воздуха). Постоянство ускорения силы тяжести многократно проверялось после Галилея. Кульминационным пунктом были исключительно точные эксперименты Роланда (Лоранда) Этвеша, выполненные между 1889 и 1908 гг. Физики считали, что работы Этвеша оказали решающее влияние на Альберта Эйнштейна, создавшего общую теорию относительности в период между 1908 и 1915 гг. Автор статьи R. Н. D i с k e, The EotvSs Experiment, Scientific American 205 (6), 84 (1961) утверждает, что гравитационное ускорение меди и свинца одинаково и этот эксперимент выполнен с точностью приблизительно до десять минус в десятой степени - [https://ufn.ru/ufn63/ufn63_2/Russian/r632f.pdf].


21.02.2024, 13:05 Дудин Александр Тимофеевич
Отзыв: Уважаемый Улугбек Товфикович! Эксперимент Этвеша, - это абсолютно не о чём, это глубокое заблуждение. Это равносильно стоять с отвесом и ждать, когда он отклонится от центробежной силы вращения Земли. Гравитационное ускорение Земли 9,82 м/с^2, а центробежное ускорение на экваторе 0,03 м/с^2. А теперь Вы попробуйте найти разницу в ускорениях, в лаборатории Принстонского университета, по линии с запада на восток, на расстоянии 40 см (длина коромысла), гравитационного ускорения и центробежного ускорения, и как от этой разницы изменяться ускорения грузов? И всё это зафиксировать зрительно? Надо быть очень благодарным Этвешу, что он не исказил результат и сказал, что он нулевой. А ещё Этвеш ошибочно считал, что между центробежной силой и гравитационной силой существует строгая пропорциональность. Если бы было так, то все небесные тела вращались вокруг центрального тела по круговой орбите. Эти силы не имеют строгой пропорциональности и изменяются по разным законам в зависимости от расстояния. Поэтому орбиты небесных тел эллипсоидные. Предлагаю ознакомиться с работами: Ускорения тел на орбите вокруг центрального тела. Часть 1 https://sci-article.ru/stat.php?i=1653847153 ; Ускорения тел на орбите вокруг центрального тела. Часть 2 https://sci-article.ru/stat.php?i=1653934276 Если у Вас есть возможность, на новом уровне провести эксперимент Этвеша, то в этом случае могу поделиться своим мнением. С уважением А.Т. Дудин.


21.02.2024, 19:08 Цорин Борис Иосифович
Отзыв: Улугбек Товфикович, чтоб Вам не пришлось знакомиться с еще "работами" Дудина, сразу пересказываю их: он берет известную формулу, подставляет в нее вместо средней круговой скорости, которая в этой формуле должна быть, среднюю скорость движения по орбите, получает мнимое несовпадение двух величин и делает далеко идущие выводы.


22.02.2024, 8:32 Суэтин Яков Аркадьевич
Отзыв: Ещё в далёкие школьные годы, когда мой жизненный опыт в бросание камней и других предметов, пришёл в противоречие с высказыванием учительницы о том, что если не учитывать сопротивление воздуха, то пушинка и камень будут иметь одну и ту же скорость падения, я хорошо выяснил для себя этот вопрос. Поэтому сейчас, когда с большой точностью доказано соответствие гравитационной и инертной масс тела, разгоревшаяся дискуссия в научном (!) журнале вызывает у меня неподдельное удивление. Скорость падения любых тел (лёгких или тяжёлых) под действием силы гравитации и без сопротивления среды, равны. Если тело, на которое они падают, закреплено, то они достигнут его одновременно. Если не закреплено, то тяжелое тело достигнет его быстрее. Так как в этом случае незакреплённое тело будет двигаться навстречу падающим телам (согласно ЗВТ) и тем быстрее, чем тяжелее сближающееся с ним тело. Например, Земля не является закреплённым телом и пушинка, падая на неё, одновременно и притягивает её к себе, уменьшая при этом время своего сближения с ней.


22.02.2024, 10:37 Дудин Александр Тимофеевич
Отзыв: 21.02.2024, 19:08 Цорин Борис Иосифович Отзыв: Улугбек Товфикович, чтоб Вам не пришлось знакомиться с еще "работами" Дудина, сразу пересказываю их: он берет известную формулу, подставляет в нее вместо средней круговой скорости, которая в этой формуле должна быть, среднюю скорость движения по орбите, получает мнимое несовпадение двух величин и делает далеко идущие выводы. О таких проблемах в народе говорят: «К каждой бочке затычка». "Чем больше масса планеты, тем больше период" - еще и математику не знаете. При фиксированных полуосях орбит чем больше масса, тем МЕНЬШЕ период. Квадрат периода обратно пропорционален сумме массы планеты и Солнца (или иных взаимодействующих тел). T^2=4*(Pi^2)*(a^3)/(G*(M+m)). 20.02.2024, 6:59 Цорин Борис Иосифович С такими «шедеврами» к преподаванию в школе математики, физики и астрономии Вас точно допускать нельзя. Влезли в систему, заменили один параметр, оставили другой без изменения, и получили период, какой хотели? Где ж Вы всему этому учились? С уважением А.Т. Дудин.


22.02.2024, 17:26 Цорин Борис Иосифович
Отзыв: Яков Аркадьевич, заметьте, при одновременном броске "если не закреплено, то тяжелое тело достигнет его быстрее" неверно. Только при поочередном.


23.02.2024, 14:14 Дудин Александр Тимофеевич
Отзыв: Уважаемый Яков Аркадьевич! Спасибо, что проявили интерес. Все мои работы противоречат Вашей точке зрения, поэтому ещё раз возражать не имеет смысла. Приглашаю Вас ознакомиться со всеми работами. С уважением А.Т. Дудин.


23.02.2024, 20:44 Алиханова Дилара Искандеровна
Отзыв: На комментарий автора от 17.02.2024, 7:29. Неужели автору не очевидно, что если молоток и перо имеют равные ускорения в гравитационном поле Луны (1,623255738593231 м/с^2 как он сам определил), то и в каждой точки дистанции сближения с Луной скорости их будут также равны. Остаётся только ответить на вопрос - какое расстояние до сближения пройдёт каждое из этих тел. Если какому-то из тел придётся пройти до сближения больший путь, чем другому, то можно будет условно сказать, что оно "падает медленней", так как потратит на этот путь больше времени (но помним, что скорости падения у них равны). Опять воспользуемся расчётом автора. В гравитационном поле молотка Луна развивает ускорение 2,916147331740634*10^-23 м/с^2, а в гравитационном поле пера - 0,1151971890525798 *10^-23 м/с^2. Пока всё правильно, но здесь автор допускает принципиальную ошибку, считая что и молоток, и перо в процессе своего падения отталкивают от себя Луну, в то время как согласно ЗВТ они её наоборот притягивают. Очевидно, что тяжелый молоток притягивает Луну сильнее, чем легкое перо (2,916147331740634*10^-23 больше 0,1151971890525798 *10^-23) и близится с ней быстрее (условно говоря "упадёт" быстрее). Такое непонимание ЗВТ приводит автора и к ошибочному выводу о том, тела с одинаковой массой не притягиваются друг к другу. Такая фраза встречается у него в нескольких статьях.


26.02.2024, 4:42 Дудин Александр Тимофеевич
Отзыв: Уважаемая Дилара Искандеровна! «Опять воспользуемся расчётом автора. В гравитационном поле молотка Луна развивает ускорение 2,916147331740634*10^-23 м/с^2 …» Масса Луны 7,3477 *10^22 кг Ускорение Луны 2,916147331740634*10^-23 м/с^2 Находим силу притяжения Луны молотком: F(м) = M*a = 7,3477 *10^22 кг * 2,916147331740634*10^-23 м/с^2 = 2,142697574943066 Н F(м) = 2,142697574943066 Н Находим силу притяжения молотка Луной: Масса молотка 1,32 кг. Ускорение, с каким Луна притягивает молоток: 1,623255738593231 м/с^2 F(л) = m*a(л) = 1,32 кг * 1,623255738593231 м/с^2 = 2,142697574943065 Н F(л) = 2,142697574943065 Н F(м) = 2,142697574943066 Н F(л) = F(м) Весь закон всемирного тяготения (ЗВТ) сводиться к определению одной из сил третьего закона И.Ньютона. Если массы и ускорения равны, то: Ма = Ма Ма - Ма = 0 М(а-а) = 0 Как видим, одинаковые массы имеют одинаковые ускорения и притягиваться не будут. С уважением А.Т. Дудин.


26.02.2024, 16:46 Цорин Борис Иосифович
Отзыв: {1} "Если массы и ускорения равны, то: Ма = Ма, Ma - Ma=0" - сила действия и сила противодействия равны по модулю, но направлены в разные стороны. Почему Вы векторные величины вычитаете, как скалярные, если они направлены в разные стороны? {2} Ну или, если слова "векторные" и "скалярные" Вам не знакомы (хотя это школьный курс), вот Вам аналогия к этому Вашему "рассуждению". Пусть два автомобиля едут навстречу друг другу со скоростью V каждый. V=V. V-V=0. Ура, они не столкнутся, потому что не сближаются, так, что ли? )


26.02.2024, 17:20 Цорин Борис Иосифович
Отзыв: Что касается того, какое тело будет падать быстрее, есть четыре основных варианта. Точнее, есть еще много вариантов, различающихся тем, какое тело в какую точку Земли падает, но их мы не учитываем, ведь и Земля неравномерна, и космические тела с разной стороны влияют. Так что рассматриваем легкое и тяжелое тело, падающие из одной точки пространства на одну точку Земли. Воздух, естественно, в месте падения убираем полностью. Вариант 1: они падают одновременно. Тогда они упадут одновременно, потому что насколько бы сильно они ни притягивали Землю, они ее притягивают вместе. Вариант 2: одно тело падает, потом падает второе. Тогда второе будет падать быстрее, и неважно, легче оно или тяжелее. Ведь его будет притягивать вниз не только Земля, но и уже лежащее первое тело. Вариант 3: одно тело падает на лежащее второе, потом второе поднимается и падает на первое (прямо рассказ об алкашах получается). Тогда время падения будет одинаково, потому что сумма сил притяжения трех тел совпадает. Вариант 4: тело появляется из ниоткуда, падает, исчезает в никуда, затем так же поступает второе тело. Тогда быстрее упадет тяжелое тело (а вычитание имени Дудина - бред), но в условиях открытия телепортации нас это уже вряд ли будет интересовать.


Оставить комментарий


 
 

Вверх